Что такое верные неравенства 2 класс: Верные и неверные равенства и неравенства — урок. Математика, 1 класс.

Содержание

Верные равенства или верные неравенства. Что такое равенство? Первый признак и принципы равенства

Сравним две точки и единицу, число 2 правее на числовом луче, чем число 1. Ставим знак >. Это неравенство. (рис. 13)

Рис. 13

Вставьте в окошко число, чтобы полученное равенство и неравенство стали верными.

Это неравенство. Посмотрим на числовой луч. Раз мы ищем число меньше, чем число 7, значит оно должно быть левее числа 7 на числовом луче. (рис. 14)

Рис. 14

В окошко можно вставить несколько чисел. Сюда подходят числа 0, 1, 2, 3, 4, 5, 6. Любое из них можно подставить в окошко и получить несколько верных неравенства. Например, 5

На числовом луче найдём числа, которые будут меньше 5. (рис. 15)

Рис. 15

Это числа 4, 3, 2, 1, 0. Следовательно, любое из этих чисел можно подставить в окошко, мы получим несколько верных неравенств. Например, 5 >4, 5 >3

В можно подставить только одно число 8.

На данном уроке мы познакомились с математическими понятиями: «равенство» и «неравенство», научились правильно расставлять знаки сравнения, потренировались сравнивать группы фигур с помощью составления пар и сравнивать числа с помощью числового луча, что поможет в дальнейшем изучении математики.

Список литературы

  1. Александрова Л.А., Мордкович А.Г. Математика 1 класс. — М: Мнемозина, 2012.
  2. Башмаков М.И., Нефедова М.Г. Математика. 1 класс. — М: Астрель, 2012.
  3. Беденко М.В. Математика. 1 класс. — М7: Русское слово, 2012.
  1. Igraem.pro ().
  2. Slideshare.net ().
  3. Iqsha.ru ().

Домашнее задание

1. Какие знаки сравнения вы знаете, в каких случаях они используются? Запишите знаки сравнения чисел.

2. Сравните количество предметов на рисунке и поставьте знак «» или «=».

3. Сравни числа, поставив знак «» или «=».

Два числовых математических выражения, соединенные знаком «=» называют равенством.

Например: 3 + 7 = 10 — равенство.

Равенство может быть верным и неверным.

Смысл решения любого примера состоит в том, чтобы найти та­кое значение выражения, которое превращает его в верное равенство.

Для формирования представлений о верных и неверных равенствах в учебнике 1 класса используются примеры с окошком.

Например:

Методом подбора ребенок находит подходящие числа и проверяет верность равенства вычислением.

Процесс сравнения чисел и обозначение отношений между ними с помощью знаков сравнения приводит к получению неравенств.

Например: 5 4 — числовые неравенства

Неравенства также могут быть верными и неверными.

Например:

Методом подбора ребенок находит подходящие числа и проверяет верность неравенства.

Числовые неравенства получаются при сравнении числовых выражений и числа.

Например:

При выборе знака сравнения ребенок вычисляет значение выражения и сравнивает его с заданным числом, что отражается в выборе соответствующего знака:

10-2>7 5+К7 7 + 3>9 6-3 = 3

Возможен другой способ выбора знака сравнения — без ссылки на вычисления значения выражения.

Наппимеп:

Сумма чисел 7 и 2 будет заведомо больше, чем число 7, значит, 7 + 2 > 7.

Разность чисел 10 и 3 будет заведомо меньше, чем число 10, значит, 10 — 3

Числовые неравенства получаются при сравнении двух числовых выражений.

Сравнить два выражения — значит сравнить их значения. Например:

При выборе знака сравнения ребенок вычисляет значения выражений и сравнивает их, что отражается в выборе соответствующего знака:

Возможен другой способ выбора знака сравнения — без ссылки на вычисления значения выражения. Например:

Для постановки знаков сравнения можно провести такие рассуждения:

Сумма чисел 6 и 4 больше суммы чисел 6 и 3, поскольку 4 > 3, значит, 6 + 4 > 6 + 3.

Разность чисел 7 и 5 меньше, чем разность чисел 7 и 3, поскольку 5 > 3, значит, 7 — 5

Частное чисел 90 и 5 больше, чем частное чисел 90 и 10, поскольку при делении одного и того же числа на число большее, частное получается меньшее, значит, 90: 5 > 90:10.

Для формирования представлений о верных и неверных равенствах и неравенствах в новой редакции учебника (2001) используются задания вида:

Для проверки используется метод вычисления значения выражений и сравнения полученных чисел.

Неравенства с переменной практически не используются в последних редакциях стабильного учебника математики, хотя в более ранних изданиях они присутствовали. Неравенства с переменными активно используются в альтернативных учебниках математики. Это неравенства вида:

 + 7 2;  > 0;  > О

После введения буквы для обозначения неизвестного числа такие неравенства приобретают привычный вид неравенства с переменной:

а + 7>10; 12-d

Значения неизвестных чисел в таких неравенствах находятся методом подбора, а затем подстановкой проверяется каждое подобранное число. Особенность данных неравенств состоит в том, что могут быть подобраны несколько чисел, подходящих к ним (дающих верное неравенство).

Например: а + 7 > 10; а = 4, а = 5 , а = 6 и т. д. — количество значений для буквы а бесконечно, для данного неравенства подхо­дит любое число а > 3; 12 — d

В случае бесконечного множества решений или большого количества решений неравенства ребенок ограничивается подбором нескольких значений переменной, при которых неравенство является верным.

«Равенство» — это тема, которую ученики проходят еще в начальной школе. Сопутствует ей также ей «Неравенства». Эти два понятия тесно взаимосвязаны. Кроме того, с ними связывают такие термины, как уравнения, тождества. Итак, что такое равенство?

Понятие равенства

Под этим термином понимают высказывания, в записи которых есть знак «=». Равенства разделяются на верные и неверные. Если в записи вместо = стоит , тогда речь идет о неравенствах. Кстати, первый признак равенства говорит о том, что обе части выражения идентичны по своему результату или записи.

Кроме понятия равенства, в школе изучают также тему «Числовое равенство». Под этим высказыванием понимают два числовых выражения, которые стоят по обе стороны от знака =. К примеру, 2*5+7=17. Обе части записи равны между собой.

В числовых выражениях подобного типа могут использоваться скобки, влияющие на порядок действий. Итак, существует 4 правила, которые следует учесть при вычислении результатов числовых выражений.

  1. Если в записи нет скобок, тогда действия выполняются с высшей ступени: III→II→I. Если есть несколько действий одной категории, тогда они выполняются слева направо.
  2. Если в записи есть скобки, тогда действие выполняется в скобках, а затем с учетом ступеней. Возможно, в скобках будет несколько действий.
  3. Если выражение представлено в виде дроби, тогда вычислять нужно сначала числитель, потом знаменатель, затем числитель делится на знаменатель.
  4. Если в записи есть вложенные скобки, тогда вычисляется сначала выражение во внутренних скобках.

Итак, теперь понятно, что такое равенство. В дальнейшем будут рассмотрены понятия уравнения, тождества и способы их вычисления.

Свойства числовых равенств

Что такое равенство? Изучение этого понятия требует знания свойств числовых тождеств. Приведенные ниже текстовые формулы позволяют лучше изучить данную тему. Конечно, эти свойства больше подходят для изучения математики в старших классах.

1. Числовое равенство не будет нарушено, если в обеих его частях прибавить одно и то же число к существующему выражению.

А = В ↔ А + 5 = В + 5

2. Не будет нарушено уравнение, если обе его части умножить или разделить на одно и то же число или выражение, которые отличны от нуля.

Р = О ↔ Р ∙ 5 = О ∙ 5

Р = О ↔ Р: 5 = О: 5

3. Прибавив к обеим частям тождества одинаковую функцию, которая имеет смысл при любых допустимых значениях переменной, мы получим новое равенство, равносильное первоначальному.

F(X) = Ψ (X) F(X) + R(X) = Ψ

(X) + R(X)

4. Любое слагаемое или выражение можно перенести по другую сторону знака равенства, при этом нужно поменять знаки на противоположные.

Х + 5 = У — 20 Х = У — 20 — 5 Х = У — 25

5. Умножив или разделив обе части уравнения на одну и ту же функцию, отличную от нуля и имеющую смысл для каждого значения Х из ОДЗ, мы получим новое уравнение, равносильное первоначальному.

F(X) = Ψ(X) F(X) ∙ R(X) = Ψ(X) ∙ R(X)

F(X) = Ψ (X) F(X) : G(X) = Ψ (X) : G(X)

Приведенные правила в явной степени указывают на принцип равенства, который существует при определенных условиях.

Понятие пропорции

В математике существует такое понятие, как равенство отношений. В этом случае подразумевается определение пропорции. Если разделить А на В, то результатом будет отношение числа А к числу В. Пропорцией называют равенство двух отношений:

Иногда пропорция записывается следующим образом: A: B = C: D. Отсюда вытекает основное свойство пропорции: A * D = D * C , где A и D — крайние члены пропорции, а В и С — средние.

Тождества

Тождеством называют равенство, которое будет верно при всех допустимых значениях тех переменных, которые входят в задание. Тождества могут быть представлены как буквенные или числовые равенства.

Тождественно равными называются выражения, содержащие в обеих частях равенства неизвестную переменную, которая способна приравнять две части одного целого.

Если проводить замены одного выражения другим, которое будет равно ему, тогда речь идет о тождественном преобразовании. В этом случае можно воспользоваться формулами сокращенного умножения, законами арифметики и прочими тождествами.

Чтобы сократить дробь, нужно провести тождественные преобразования. К примеру, дана дробь. Чтобы получить результат, следует воспользоваться формулами сокращенного умножения, разложением на множители, упрощением выражений и сокращением дробей.

При этом стоит учесть, что данное выражение будет тождественным тогда, когда знаменатель не будет равен 3.

5 способов доказать тождество

Чтобы доказать равенство тождественное, нужно провести преобразование выражений.

I способ

Необходимо провести равносильные преобразования в левой части. В результате получается правая часть, и можно говорить о том, что тождество доказано.

II способ

Все действия по преобразованию выражения происходят в правой части. Итогом проделанных манипуляций является левая часть. Если обе части идентичны, то тождество доказано.

III способ

«Трансформации» происходят в обеих частях выражения. Если в результате получатся две идентичные части, тождество доказано.

IV способ

Из левой части вычитается правая. В результате равносильных преобразований должен получиться нуль. Тогда можно говорить о тождественности выражения.

V способ

Из правой части вычитается левая. Все равносильные преобразования сводятся к тому, чтобы в ответе стоял нуль. Только в таком случае можно говорить о тождественности равенства.

Основные свойства тождеств

В математике зачастую используют свойства равенств, чтобы ускорить процесс вычисления. Благодаря основным алгебраическим тождествам процесс вычисления некоторых выражений займет считанные минуты вместо долгих часов.

  • Х + У = У + Х
  • Х + (У + С) = (Х + У) + С
  • Х + 0 = Х
  • Х + (-Х) = 0
  • Х ∙ (У + С) = Х∙У + Х∙С
  • Х ∙ (У — С) = Х∙У — Х∙С
  • (Х + У) ∙ (С + Е) = Х∙С + Х∙Е + У∙С + У∙Е
  • Х + (У + С) = Х + У + С
  • Х + (У — С) = Х + У — С
  • Х — (У + С) = Х — У — С
  • Х — (У — С) = Х — У + С
  • Х ∙ У = У ∙ Х
  • Х ∙ (У ∙ С) = (Х ∙ У) ∙ С
  • Х ∙ 1 = Х
  • Х ∙ 1/Х = 1, где Х ≠ 0

Формулы сокращенного умножения

По своей сути формулы сокращенного умножения являются равенствами. Они помогают решить множество задач в математике благодаря своей простоте и легкости в обращении.

  • (А + В) 2 = А 2 + 2∙А∙В + В 2 — квадрат суммы пары чисел;
  • (А — В) 2 = А 2 — 2∙А∙В + В 2 — квадрат разности пары чисел;
  • (С + В) ∙ (С — В) = С 2 — В 2 — разность квадратов;
  • (А + В) 3 = А 3 + 3∙А 2 ∙В + 3∙А∙В 2 + В 3 — куб суммы;
  • (А — В) 3 = А 3 — 3∙А 2 ∙В + 3∙А∙В 2 — В 3 — куб разности;
  • (Р + В) ∙ (Р 2 — Р∙В + В 2) = Р 3 + В 3 — сумма кубов;
  • (Р — В) ∙ (Р 2 + Р∙В + В 2) = Р 3 — В 3 — разность кубов.

Формулы сокращенного умножения зачастую применяются, если необходимо привести многочлен к привычному виду, упростив его всеми возможными способами. Представленные формулы доказываются просто: достаточно раскрыть скобки и привести подобные слагаемые.

Уравнения

После изучения вопроса, что такое равенство, можно приступать к следующему пункту: Под уравнением понимается равенство, в котором присутствуют неизвестные величины. Решением уравнения называют нахождение всех значений переменной, при которых обе части всего выражения будут равны. Также встречаются задания, в которых нахождение решений уравнения невозможно. В таком случае говорят, что корней нет.

Как правило, равенства с неизвестными в качестве решения выдают целые числа. Однако возможны случаи, когда корнем являются вектор, функция и другие объекты.

Уравнение является одним из важнейших понятий в математике. Большинство научных и практических задач не позволяют измерить или вычислить какую-либо величину. Поэтому необходимо составлять соотношение, которое удовлетворит все условия поставленной задачи. В процессе составления такого соотношения появляется уравнение или система уравнений.

Обычно решение равенства с неизвестным сводится к преобразованию сложного уравнения и сведению его к простым формам. Необходимо помнить, что преобразования нужно проводить относительно обеих частей, в противном случае на выходе получится неверный результат.

4 способа решить уравнение

Под решением уравнения понимают замену заданного равенства другим, которое равносильно первому. Подобная подмена известна как тождественное преобразование. Чтобы решить уравнение, необходимо воспользоваться одним из способов.

1. Одно выражение заменяется другим, которое в обязательном порядке будет тождественно первому. Пример: (3∙х+3) 2 =15∙х+10. Это выражение можно преобразовать в 9∙х 2 +18∙х+9=15∙х+10.

2. Перенесение членов равенства с неизвестным из одной стороны в другую. В таком случае необходимо правильно менять знаки. Малейшая ошибка сгубит всю проделанную работу. В качестве примера возьмем предыдущий «образец».

9∙х 2 + 12∙х + 4 = 15∙х + 10

9∙х 2 + 12∙х + 4 — 15∙х — 10 = 0

3. Перемножение обеих частей равенства на равное число или выражение, которые не равняются 0. Однако стоит напомнить, что если новое уравнение не будет равносильным равенству до преобразований, тогда количество корней может существенно измениться.

4. Возведение в квадрат обеих частей уравнения. Этот способ просто замечательный, особенно когда в равенстве есть иррациональные выражения, то есть и выражение под ним. Тут есть один нюанс: если возвести уравнение в четную степень, тогда могут появиться посторонние корни, которые исказят суть задания. И если неправильно извлечь корень, тогда смысл вопроса в задаче будет неясен. Пример: │7∙х│=35 → 1) 7∙х = 35 и 2) — 7∙х = 35 → уравнение будет решено верно.

Итак, в этой статье упоминаются такие термины, как то уравнения и тождества. Все они происходят от понятия «равенство». Благодаря различного рода равносильным выражениям решение некоторых задач в значительной мере облегчено.

«Равенства и неравенства» Уравнения 2 класс

Математика

Класс: 2

Дата:

Тема подраздела/навыки:

Равенства и неравенства. Уравнения.

ФИО учителя: Суесенова А.Т.

Цели обучения (ЦО) из долгосрочного плана

1.2.2.1 распознавать равенство, неравенство, уравнение; различать верные и неверные равенства;
1.5.2.2 использовать знаки «+», «-», «≠», «=», «>», «<»;

Цель урока:

Различать равенство, неравенство;

Определять верные и неверные равенства;

Применять знаки «+», «-», «≠», «=», «>», «<»;

Языковые цели

 

Основные термины и словосочетания:

Верное и неверное равенство знаки «больше», «меньше», «равно»;

Вопросы для обсуждения:

Какой знак используется для обозначения равенства?

Какой знак используется для обозначения неравенства?

Как отличить верное равенство от неверного?

Формирование ценностей

Сотрудничество, уважение, образование в течение всей жизни.

Критерии оценивания

Определяет равенство, неравенство; Находит верные, неверные равенства;

Различает верные, неверные равенства, используя знаки «+», «-», «≠», «=», «>», «<»;

Ресурсы

Слайд, карточки с заданиями, интерактивная доска.

Предварительные знания

Знание знаков «=», «>», «<».

Запланированные периоды урока

Методы обучения

Методы оценивания

Способы дифференциации

Начало урока

 

Психологический настрой.

Я рада видеть ваши лица, ваши улыбки и думаю, что этот день принесёт вам радость, общение друг с другом. Сядьте удобно, закройте глаза и повторяйте за мной: «Я в школе, я на уроке. Я радуюсь этому. Внимание мое растет. Я как разведчик, все замечу. Память моя крепка. Голова мыслит ясно. Я хочу учиться. Я готов к работе. Я работаю. Молодцы ребята!».

Актуализация.

«И» Сравни числа.

12 9 18 11

15 8 9 12

8 12 17 19

5 9 13 17

Целеполагание.

И «Выражения вычисляй — тему урока узнавай!»

Чтобы узнать тему нашего урока, мы должны решить примеры и расположить числа в порядке возрастания. (Слайд)

2+2= В 2+1= А

1+1= Р 3+3= Е

5+2= Н 7+1= И

0+1= С 4+1= Н

5+4= Е

1 2 3 4 5 6 7 8 9

С Р А В Н Е Н И Е

Назовите тему нашего урока?

Мы будем составлять и решать равенства и неравенства.

 

Форма оценивания: Самооценивание по эталону

Дифференциация по способу «Диалог и поддержка» (оказывается помощь учителя учащимся, испытывающим затруднения)

Середина урока

Задание 1(И) «Подбери примеры!»

Соотнеси равенство и неравенство по кружочкам.

5+3=4+4 6+5≠5+5

3+3=5+1 3+9≠6+8

Задание 2. (П) «Расставь знаки»

Поставь знаки «+» или « –».

6 1<7 7 1>5 5

7 2=5 5 1=7 1

7 3>9 9 3<7 1

Критерий: Различает верные неверные равенства используя знаки «+», «-».

Дескриптор: — различает неравенства и ставит знак «+» — различает неравенства и ставит знак «-» — различает равенства и ставит знак «+» — различает равенства и ставит знак «-»

Игровое упражнение « Вагончики»

У каждого учащегося карточка с выражением (равенство или неравенство). Перед доской стоят 2 машиниста и держат в руках листочки, на которых написано: «Равенства» и «Неравенства» Дети читают свое выражение и присоединяются к вагончикам.

Что называется равенством?

Что называется неравенством?

(И) Задание 3. «Найди ошибку».

Решите примеры. Найдите ошибки.

Сколько неверных равенств ты нашел?

9+3=5+6 8+5≠7+7

14-5≠10-5 6+5=7+4

10+2=7+4 5+8=5+5

Критерий: различает верные, неверные равенства.

Дескрипторы:

— решает примеры;

— находит неверные равенства;

Дополнительное задание:

У Айгуль 3 красных цветка, а у Светы 4 розовых. У кого больше? Составьте и назовите выражение. (Неравенство: 3 ≠ 4.)

Критерий: определяет равенство и неравенство.

Дескрипторы:

-читает задачу и составляет краткую запись

-составляет выражение со знаком «-»

— определяет неравенство.

Физминутка:

Мы решали, мы решали.

Что-то очень мы устали.

Мы сейчас потопаем,

Ручками похлопаем

Раз присядем,

Быстро встанем,

Улыбнемся, тихо сядем.

Задание 4. (Г) «Попробуй-ка!»

Задание уровня А: Соединить стрелками числовые и буквенные выражения, равенства и неравенства

3+а равенство

8-4 неравенство

4+5=9 буквенное выражение

3+5>6 числовое выражение

Критерий: находит верные, неверные равенства;

Дескрипторы:

-соотносит выражение со словом «равенство»

-соотносит выражение со словом «неравенство»

-соотносит выражение со словом «буквенное выражение»

— соотносит выражение со словом « числовое выражение

Задание уровня В: Зачеркни неверные равенства.

8+4=13 12-5 =9

15-2 =13 16-10 =5

9+2 =11 12+6=17

15+8=22 13+4=17

Критерий: находит неверные равенства;

Дескрипторы: — зачеркивает неверные равенства.

Задание уровня С: Сравните значения числовых выражений, определите их вид.

6+8*9+4 5+8*6+3

17-3*14+2 7+4*11+6

28-4*20+4 25-6*18-4

Критерий: определяет равенство, неравенство;

Дескриптор: сравнивает значение выражения и ставит знак «=», «>», «<».

Форма оценивания:

Самооценивание по критерию и дескрипторам

Прием «сигнал рукой».

 Форма оценивания:

Взаимооценивание по шаблону

Форма оценивания

Взаимооценивание по критерию и дескрипторам.

Форма оценивания:

Самооценивание. Прием: «Светофор»

Форма оценивания : Взаимооценивание

Прием «Две звезды одно пожелание»

Дифференциация по способу «Диалог и поддержка» (оказывается помощь учителя учащимся, испытывающим затруднения)

Дифференциация по способу «Диалог и поддержка» (оказывается помощь учителя учащимся, испытывающим затруднения)

Дифференциация по способу «Диалог и поддержка» (оказывается помощь учителя учащимся, испытывающим затруднения)

«Темп»

Некоторые учащиеся решают быстрее, для них подготовлены дополнительное задание.

Дифференциация по способу:

«Задание»

Учащимся даются задания разного уровня.

Конец урока

Для закрепления урока проведу игру. Разделю класс на группы по 4 человек. По команде «Равенство!» участники игры внутри групп должны разделиться на равные подгруппы. По команде «Неравенство!» дети в группах делятся на неравные подгруппы.

«Повторение – мать ученье»

Что называется равенством?

Что называется неравенством?

Какой знак используется для обозначения  равенства?

Какой знак используется для обозначения неравенства?

Рефлексия:

 

Рефлексия учителя по проведенному уроку

1) Количество учащихся достигших и не достигших ЦО?

2) Если не достигли, то почему?

3) Какие отклонения были от плана урока, почему?

 

Общая оценка

Какие приемы и методы были наиболее эффективны для достижения цели обучения?

Что могло бы способствовать улучшению урока?

 Чему нужно уделить внимание на следующих уроках (достижения и затруднения отдельных учеников, класса)?

Неравенства. Верные и неверные неравенства. Математика (2 класс)

Математика, 2 класс

Тема урока: «Неравенства. Верные и неверные неравенства»

-познакомить с понятием «неравенство» и видами неравенств;

-совершенствовать вычислительные умения и навыки;

— совершенствовать умение решать задачи.

1

2

3

4

4.1

4.2

4.3

5

5.1

5.2

6

6.1

6.2

7

7.1

7.2*

8

9

10

Орг. момент

Сегодня наш урок будет необычным, мы отправимся в путешествие.

А на каком виде транспорта, отгадайте:

Чудо-птица, алый хвост,

Полетела в стаю звёзд.

Проверим готовность к уроку:

-Руки?

-Ноги?

-Локти?

-Спина?

Проверка домашнего задания

От жителей далёкой планеты «Решай-считай» пришло к нам послание. Проверив д/з, мы попробуем его расшифровать.

-Сравнивая числовые выражения, мы поставили знак «равно». У нас при записи получились равенства.

— А что значит «неравенство»? И какие бывают неравенста?

Постановка цели и задач урока

Тема нашего урока : «Неравенства. Верные и неверные неравенства»

Какую цель мы поставим перед собой?

Что поможет достигнуть эту цель?

В конце урока сигнальными карточками вы оцените свою работу на уроке:

работал в полную силу,

работал хорошо, но были ошибки

надо подтянуться

Устный счёт

Много заданий у нас впереди.

Ну, а сейчас тренировки нужны.

Для достижения цели нам понадобятся не только математические знания, но и умение работать вместе, прийти на помощь друг другу в трудную минуту.

Решив примеры, вы расшифруете девиз нашего урока:

80-15 22+18 93-6 90-40 19+7 42+50 62-20

65 40 87 50 26 92 42

Каждый парашют должен приземлиться в заданной точке-ответе

Начинаем тренировку,

Чтобы сильным стать и ловким.

Круговые примеры «Центрифуга»

(это устройство, создающее перегрузки под действием центробежной силы)

-10 -42

-4

86

18+

36+ 2+

Сравните числа и числовые выражения:

1гр 2гр 3гр

85…43 28…57 14…54

79-20…50 35+5…60 46+4…30

38…53+7 96…50-30 48…90-30

Неравенства. Верные и неверные неравенства

Подготовка закончилась. Мы готовы к полёту.

Любой космический маршрут

Открыт для тех, кто любит труд.

Откройте наши путеводители (ученик стр. 90), приготовьте бортовые журналы (тетради).

В предыдущем задании мы сравнивали

-числа,

-числа и числовые выражения.

Между числами 85…43 28…57

стоят знаки < или >(нет знака =). Значит это неравенство. А мы уже встречались с неравенствами?

Рассмотрите следующие неравенства

79-20…50 35+5…60

Что можно о них сказать, правильно ли стоит знак? Значит это верное неравенство. В математике слово «правильно» заменили словом «верно». А если знак изменить? Будет ли верным это неравенство?

Значит это неверное неравенство.

стр.90 №2

-выпишите верные неравенства.

Какие неравенства называются верными? (верно поставлен знак)

ФИЗКУЛЬТМИНУТКА

Космонавты всегда занимаются спортом и делают зарядку. Даже в полёте они находят время на спортивные упражнения:

В небе плавает луна.

В облака зашла она.

Раз, два, три, четыре, пять-

Можем мы луну достать.

Шесть, семь, восемь, девять, десять-

И пониже перевесить.

Десять, девять, восемь, семь-

Чтоб луна светила всем.

стр.91 №4

-вставить недостающие числа, чтобы неравенства были верными

(В тетрадь записываем 1 нерав-во, устно проговариваем с другими числами)

Решение задач изученных видов

Любой космонавт должен хорошо уметь решать задачи.

стр.91 №6

Выполняем необходимые расчеты в бортовых журналах

ФИЗКУЛЬТМИНУТКА

Чтоб глаза твои зорче были,

Чтоб очки тебе не носить,

Эти лёгкие движения

Предлагаю повторить!

упр. 1

круговые движения глазами по часовой стрелке- 4 раза,

против часовой стрелки-4 раза

упр. 2

вверх, вниз-4 раза вправо, влево-4 раза

упр. 3

откинуться на спинку стула, сделать глубокий вдох, наклониться вперёд-выдох -4 раза

стр.91 №5

кратк. усл.

Было-92арб. Было-92 арб Было-?

Пр.-46 арб. Пр-? Пр-46 арб

Ост.- ? Ост-46 арб Ост- 46 арб

Закрепление пройденного материала

Составьте верное неравенство:

-с числами15 и 2;

-с числом и числовым выражением

45 и 15+25

Жители планеты «Решай-считай» предлагают вам выполнить самостоятельно задание:

сравните числа и числовые выражения

39-23…20

47…69+2

37+ 13…42+28

Оцените правильность выполнения задания

стр. 90 №3 1 столбик

Рассмотрите внимательно числовые выражения. Кто может не вычисляя сравнить? Какие неравенства получились верные или неверные?

Это интересно. Математика в числах

*12 апреля 1961 г Юрий Алексеевич Гагарин на космической ракете «Восток»

поднялся в космос. Его полёт продолжался 108 минут.

* К полёту космонавтов готовят от 6 до 12 лет

*В космическом пространстве побывало 3 наших земляка (соотечественника):

Пётр Климук, Владимир Ковалёнок, Олег Новицкий.

О. Новицкий пробыл на борту космической станции 4 месяца и приземлился 16 марта 2013г.

*Сколько планет в солнечной системе?

8-Меркурий, Венера, Земля, Марс, Юпитер, Сатурн, Уран, Нептун.

(До 2006 года считалось 9. Плутон не входит в список планет)

Итог урока. Рефлексия

Кем мы были на уроке?

Что нового узнали?

С какими неравенствами познакомились?

Цель урока достигнута?

Продолжите высказывания.

Оцените свою работу на уроке.

Д/З стр91 №1

Ракета

-На месте!

-На месте!

-У края!

-Прямая!

21-12=30-21

82-34=60-12

Оцените правильность выполнения Д/З на линеечке самооценки

— Познакомить с понятием «неравенство» и видами неравенств;

-совершенствовать вычислительные умения и навыки;

-совершенствовать умение решать задачи

Ученики решают примеры и прикрепляют карточки с ответами.

Переворачивают карточки-ответы, читают девиз:

«Один за всех и все за одного»

К доске выходят ученики и вставляют пропущенные знаки

Да, когда сравнивали числа и числовые выражения. И при сравнении ставили знаки < или >

Нет

1 ученик у доски

34 <49

10 >25-20

72+(6+2)> 72

Остальные ученики выполняют задание в тетради

…+1< 5(Ч:0,1,2,3)

7-… >3 (Ч:0,1,2,3)

10-…> 5 (Ч:0,1,2,3,4)

Задача

1 нед.=7 дн.

2 нед.=14дн.

14+2=16дн.

Ответ: 16 дней.

Задача

92-46=46(арб.)

Составление обратных задач:

1) В магазине было 92 арбуза. Сколько арбузов продали, если в магазине осталось 46 арбузов?

2)Сколько арбузов было в магазине, если за день продали 46 арбузов и столько же арбузов осталось?

15 >2

45 >15+25

39-23 < 20

47 < 69+2

42+28>37+ 13

51-18 >51-19

38+41 >38+40

Космонавтами Исследователями

2 класс. ПНШ. Методическая разработка урока по математике. ЧИСЛОВЫЕ РАВЕНСТВА И ЧИСЛОВЫЕ НЕРАВЕНСТВА

Урок 4
ЧИСЛОВЫЕ РАВЕНСТВА И ЧИСЛОВЫЕ НЕРАВЕНСТВА

Цели: познакомить с понятием «круглые» двузначные числа; учить читать и записывать «круглые» двузначные числа, считать десятками; развивать умение анализировать и сравнивать.

Ход урока

I. Организационный момент.

II. Устный счет.

1. Расставьте числа в пустые клетки квадрата так, чтобы по всем направлениям сумма чисел была равна 15.

2. Разгадайте, как связаны числа и рисунки, и запишите верные равенства:

3. Назовите признаки, по которым изменяются фигуры в каждом ряду.

– Выберите фигуру, которой можно продолжить каждый ряд.

4. Решите задачу.

Мальвина загадывала Буратино и Пьеро загадки.

Буратино отгадал 5 загадок, а Пьеро – 12. Кто отгадал загадок больше и на сколько?

– Сколько всего загадок отгадали Буратино и Пьеро?

III. Сообщение темы урока.

– Рассмотрите записи на доске:

– Сравните записи в каждом столбике. Чем они похожи? Чем отличаются?

– Сегодня на уроке мы узнаем, как называются данные математические записи, если в «окошко» вставить числа.

IV. Работа по теме урока.

1. Задание 1.

– Дополните записи так, чтобы они получились верными.

Запись:

7 = 7

7 6

9 – 4 = 5

9

5 + 3 = 3 + 5

8

7 5

7

7 4

6

7 3

5

7 2

4

7 1

3

2

1

0

2. Задание 2.

– Прочитайте математические записи слева и справа. Чем они похожи? Чем отличаются?

– Все ли записи являются верными?

– В столбике слева написаны числовые равенства. А в столбике справа написаны числовые неравенства. Почему эти записи так называются?

3. Задание 3.

– Прочитайте данные математические записи.

– Как они называются? (Это числовые равенства.)

– Выберите верные числовые равенства и запишите их.

Запись:

5 + 5 = 10

20 – 20 = 0

12 = 12

11 – 0 = 11

10 + 0 = 10

(3 + 8) +4 = 3 + (8 + 4)

– Какие знания помогли вам выполнить это задание?

4. Задание 4.

– Прочитайте данные математические записи.

– Как они называются? (Это числовые неравенства.)

– Выберите верные числовые неравенства и запишите их.

Запись:

15 10 16 14

10 8 + 4

30 20

Физкультминутка

5. Работа в парах.

– Составьте и запишите 5 верных числовых равенств и 5 верных числовых неравенств.

Взаимопроверка в парах.

6. Задание 6.

– Запишите все числа, при подстановке которых запись становится верным неравенством.

9

8

7

6

5

7. Задание 7.

– Запишите десять чисел, при подстановке которых данная запись становится верным неравенством.

10 10

10

10

10

10

10

V. Итог урока.

– Что нового узнали на уроке?

– Какие записи называют «числовыми равенствами»?

– Какие записи называют «числовыми неравенствами»?

Домашнее задание: учебник, с. 14, № 5.

ГДЗ по Математике 2 класс учебник Моро 2 часть ответы стр 103

Числовые и буквенные выражения

Номер 1

Запиши сумму и разность, произведение и частное чисел 18 и 2. 18 + 2, 18 – 2, 18 * 2, 18 : 2 – числовые выражения.

Что значит найти значение выражения?

Выполни вычисления.


Решение:

Сумма: 18 + 2 = 30;

Разность: 18 – 2 = 16;

Произведение: 18 * 2 = 36;

Частное: 18 : 2 = 9.

Найти значение выражения – значит выполнить все указанные действия в этом выражении.

Номер 2

Запиши выражения и вычисли их значения.

1) Из числа 80 вычесть сумму чисел 20 и 7.

2) К числу 12 прибавить разность чисел 11 и 3.


Решение:

1) 80 – (20 + 7) = 80 – 27 = 53

2) 12 + (11 – 3) = 12 + 8 = 20

Номер 3

1) Что нужно знать, чтобы можно было найти значения буквенных выражений: а + 5, d – 8, k + 14, c * 3, b: 2?

2) Найди значения выражений при а = 8, d = 28, k = 30, k = 10, k = 6, c = 4, b = 10.


Решение:

1) Чтобы найти значения буквенных выражений, необходимо знать числовые значения, которые подставляются вместо этих букв.

2) 8 + 5 = 13

28 – 8 = 20

30 + 14 = 44

10 + 14 = 24

6 + 14 = 20

4 * 3 = 12

10 : 2 = 5

Равенство. Неравенство. Уравнение

Номер 1

Объясни, почему верно неравенство

3 + 4 > 10 – 5. Как это можно доказать?


Решение:

Чтобы доказать верность неравенства, найдём значения выражений с обеих сторон:

3 + 4 = 7 и 10 – 5 = 5. Неравенство верно, т. к. 7 > 5.

Номер 2

Рассмотри рисунки и составь по ним верные неравенства или равенства.

 

Решение:

3 + 4 > 8 – 2, т. к. 7 > 6

5 − 2 = 8 – 5, т. к. 3 = 3

5 − 1 < 6 + 1, т. к. 4 < 7

Номер 3

Решение:

25 + 8 < 25 + 9, т. к. 33 < 34

43 − 9 < 43 – 6, т. к. 34 < 37 27 + 2 > 27 – 2, т. к. 29 > 25

64 + 3 = 3 + 64, т. к. 67 = 67

18 < 81 21 > 19

Номер 4

Что обозначает буква в записи уравнения? Что значит решить уравнение?

Реши уравнения: х – 8 = 2, 16 – х = 6, 5 + х = 5.


Решение:

Буква в уравнении означает неизвестное число. Решить уравнение – значит найти все его корни, при которых равенство становится верным.

х – 8 = 2, х – уменьшаемое, чтобы его найти, нужно к разности прибавить вычитаемое, х = 2 + 8 = 10.

Ответ: х = 10.

16 – х = 6, х – вычитаемое, чтобы его найти, нужно от уменьшаемого отнять разность, х = 16 – 6 = 10.

Ответ: х = 10.

5 + х = 5, х – слагаемое, чтобы его найти, нужно от суммы отнять другое слагаемое, х = 5 – 5 = 0.

Ответ: х = 0.

Задание на полях.


Решение:

Возможные комбинации:

6 + 9 + 1 = 16

2 + 4 + 8 + 2 = 16

9 + 7 = 16 = 16

8 + 8 = 16 = 16

7 + 4 + 5 = 16

3 + 4 + 9 = 16

Числовые неравенства и их свойства. Сложение и умножение числовых неравенств 8 класс онлайн-подготовка на Ростелеком

Числовые неравенства и их свойства. Сложение и умножение числовых неравенств.

Мы можем сравнить любые числа а и b и результат сравнения записать в виде равенства или неравенства, используя знаки =, <, >. Для произвольных чисел а и b выполняется одно и только одно из соотношений: a=b, a<b, a>b.

Пример 1. Сравним обыкновенные дроби 58 и 47.

Для этого приведем их к общему знаменателю: 58=3556; 47=3256.

Так как 35>32, то 58>47.

Пример 2. Сравним десятичные дроби 3,6748 и 3,675.

Цифры в разрядах единиц, десятых и сотых совпадают, а в разряде тысячных в первой дроби стоит цифра 4, а во второй – цифра 5. Так как 4<5, то 3,6748<3,675.

Пример 3. Сравним обыкновенную дробь 920 и десятичную дробь 0,45. Обратив дробь 920 в десятичную, получим, что 920=0,45.

Пример 4. Сравним отрицательные числа -15 и -23. Модуль первого числа меньше модуля второго. Значит, первое число больше второго, -15>-23.

В зависимости от вида числа мы использовали тот или иной способ сравнения. Но есть универсальный способ сравнения, который охватывает все случаи.

Число а больше числа b, если разность а-b – положительное число; число а меньше числа b, если разность a-b – отрицательное число. Если разность а-b = 0, то числа а и b равны.

На координатной прямой большее число изображается точкой, лежащей правее, а меньшее – точкой, лежащей левее.

Рассмотрим некоторые свойства числовых неравенств.

  • Если a>b, то b<a, если a<b, то b>a.

    Действительно, если разность a-b – положительное число, то разность b-a – отрицательное число, и наоборот.

  • Если a<b и b<c, то а<c.

    Докажем, что разность а-с – отрицательное число. Прибавим к этой разности числа b и –b и сгруппируем слагаемые:

    а-с = а-с+b-b = (а-b)+(b+c).

    По условию а<b и b<c. Поэтому слагаемые а-b и b-c – отрицательные числа. Значит, и их сумма является отрицательным числом. Следовательно, а<c.

  • Если a<b и c – любое число, то а+с<b+c.

    Преобразуем разность (а+с)-(b+c) = а-b

    По условию а<b, поэтому a-b – отрицательное число. Значит, и разность (а+с)-(b+c) отрицательна. Следовательно, a+c<b+c.

    Если к обеим частям верного неравенства прибавить одно и то же число, то получится верное неравенство.

  • Если a<b и c – положительное число, то aс<bс. Если a<b и c – отрицательное число, то aс>bc.

    Представим разность ас-bc в виде произведения: ас-bc = с(а-b).

    Так как a<b, то a-b – отрицательное число. Если с>0, то произведение с(а-b) отрицательно, и, следовательно, ас<bc. Если с<0, то произведение с(а-b) положительно, и, следовательно, ас>bc.

    Так как деление можно заменить умножением на число, обратное делителю, то аналогичное свойство справедливо и для деления.

    Если обе части верного неравенства умножить или разделить на одно и то же положительное число, то получится верное неравенство.

    Если обе части верного неравенства умножить или разделить на одно и то же отрицательное число и изменить знак неравенства на противоположный, то получится верное неравенство.

    s
  • Если а и b – положительные числа и а<b, то 1a>1b.

    Разделим обе части неравенства a<b на положительное число ab: aab<bab. Сократив дроби, получим, что 1b<1a, т.е. 1а>1b.

    Приведем пример использования рассмотренных свойств неравенств.

    Пример 5. Оценим периметр равностороннего треугольника со стороной а мм, если известно, что 54,2<a и a<54,3, и запишем результат в виде двойного неравенства.

    54,2·3 < 3a < 54,3·3,

    162,6 < 3a < 162,9.

    Значит, периметр Р данного треугольника больше 162,6 мм, но меньше 162,9 мм.

    Рассмотрим теперь, как выполняется сложение и умножение числовых неравенств.

  • Если a<b и c<d, то a+c<b+d.

    Прибавив к обеим частям неравенства a<b число с, получим а+с<b+с. Прибавив к обеим частям неравенства с<d число b, получим b+c<b+d.

    То есть а+с<b+с<b+d. Из этого следует, что a+c<b+d.

    Если почленно сложить верные неравенства одного знака, то получится верное неравенство.

  • Если a<b и c<d, где а,b,c,d – положительные числа, то ac<bd.

    Умножим обе части неравенства a<b на положительное число с, получим ac<bс. Умножив обе части неравенства c<d на положительное число b, получим bc<bd. Получим ac<bс<bd. Следовательно ac<bd.

    Если почленно перемножить верные неравенства одного знака, левые и правые части которых – положительные числа, то получится верное неравенство.

    Из этой теоремы следует, что

    Если числа а и b положительны и a<b, то an<bn, где n – натуральное число.

    Доказанные свойства используют для оценки суммы, разности, произведения и частного.

    Пример 6. Известно, что 15<x<16 и 2<y<3. Требуется оценить сумму х+у, разность х-у, произведение ху и частное х/у.

    Сложим почленно неравенства 15<x<16 и 2<y<3, получим 17<x+y<19.

    Оценим разность. Для этого умножим 2<y<3 почленно на (-1). Получим -3<-y<-2.

    Теперь сложим почленно неравенства 15<x<16 и -3<-y<-2. Получим 12<x-y<14.

    Оценим произведение ху. Перемножим почленно неравенства 15<x<16 и 2<y<3. Получим 30<xy<48.

    Оценим частное. Для этого сначала запишем неравенство для 1у. Получится 13<1y<12. Теперь перемножим почленно 15<x<16 и 13<1y<12. Получим 5<xy<8.

  • Ответы. Учебник. Часть 2 (с. 103)

    Числа от 1 до 100
    Что узнали, чему научились во 2 классе?
    Числовые и буквенные выражения

    1. Запиши сумму и разность, произведение и частное чисел 18 и 2.
    18 + 2, 18 – 2, 18 • 2, 18 : 2 – числовые выражения.
    Что значит найти значение выражения?
    Выполни вычисления.

    Найти значение выражения значит произвести вычисления в этом выражении.
    18 + 2 = 20
    18 – 2 = 16
    18 • 2 = 36
    18 : 2 = 9

    2. Запиши выражения и вычисли их значения.
    1) Из числа 80 вычесть сумму чисел 20 и 7.
    2) К числу 12 прибавить разность чисел 11 и 3.

    1) 80 – (20 + 7) = 80 – 27 = 53
    2) 12 + (11 – 3) = 12 + 8 = 20

    3. 1) Что нужно знать, чтобы можно было найти значения буквенных выражений: α + 5, d – 8, k + 14, c • 3, b : 2?
    2) Найди значения выражений при α = 8, d = 28, k = 30, k = 10, k = 6, c = 4, b = 10.

    1) Чтобы найти значения буквенных выражений нужно значть числовое значение букв в этих выражениях
    2) α = 8, 8 + 5 = 13
    d = 28, 28 – 8 = 20
    k = 30, 30 + 14 = 44
    k = 10, 10 + 14 = 24
    k = 6, 6 + 14 = 20
    c = 4, 4 • 3 = 12
    b = 10, 10 : 2 = 5

    Равенство. Неравенство. Уравнение

    1. Объясни, почему верно неравенство
    3 + 4 > 10 – 5. Как это можно доказать?

    Надо вычислить выражения справа и слева от знака неравенства: 3 + 4 = 7, 10 – 5 = 5. При счёте 7 следует после 5, поэтому 7 больше 5. Соответственно, 3 + 4 будет больше 10 – 5.

    2. Рассмотри рисунки и составь по ним верные неравенства или равенства.

    3 + 4 > 8 – 2
    5 – 2 = 8 – 5
    5 – 1 < 6 + 1

    3. 

    25 + 8 < 25 + 9
    43 – 9 < 43 – 6
    27 + 2 > 27 – 2
    64 + 3 = 3 + 64
    18 < 81
    21 > 19

    4. Что обозначает буква в записи уравнения? Что значит решить уравнение?
    Реши уравнения: х – 8 = 2, 16 – х = 6, 5 + х = 5.

    Неизвестное число, которое надо найти. Найти такое число, при подстановке которого в уравнение оно превратится в равенство.
    х – 8 = 2
    х = 2 + 8
    х = 10

    16 – х = 6
    х = 16 – 6
    х= 10

    5 + х = 5
    х = 5 – 5
    х = 0


    ЗАДАНИЕ НА ПОЛЯХ
    НАБЕРИ 16


    16 = 7 + 4 + 5 = 6 + 9 + 1 = 2 + 4 + 8 + 2 = 3 + 4 + 9

    Математика. 2 класс. Ответы к заданиям

    1 / 5 ( 5 голосов )

    Устранение неравенств — объяснения и примеры

    Что такое неравенство в математике?

    Слово неравенство означает математическое выражение, в котором стороны не равны друг другу. По сути, неравенство сравнивает любые два значения и показывает, что одно значение меньше, больше или равно значению на другой стороне уравнения.

    Как правило, для представления уравнений неравенства используются пять символов неравенства.

    Символы неравенства

    Эти символы неравенства: меньше ( <), больше (> ), меньше или равно (), больше или равно () и символ неравенства () .

    Неравенства используются для сравнения чисел и определения диапазона или диапазонов значений, которые удовлетворяют условиям данной переменной.

    Операции с неравенствами

    Операции с линейными неравенствами включают сложение, вычитание, умножение и деление. Общие правила этих операций показаны ниже.

    Хотя мы использовали символ <для иллюстрации, следует отметить, что те же правила применяются к>, ≤ и ≥.

    • Символ неравенства не меняется при добавлении одного и того же числа к обеим сторонам неравенства.Например, если a
    • Вычитание обеих частей неравенства на одно и то же число не меняет знака неравенства. Например, если a
    • Умножение обеих сторон неравенства на положительное число не меняет знака неравенства. Например, если a
    • Разделение обеих сторон неравенства на положительное число не меняет знака неравенства. Если a
    • Умножение обеих сторон уравнения неравенства на отрицательное число изменяет направление символа неравенства.Например, если a b *
    • Аналогичным образом, разделение обеих сторон уравнения неравенства на отрицательное число изменяет символ неравенства. Если a b / c

    Как решить неравенства?

    Подобно линейным уравнениям, неравенства могут быть решены с помощью аналогичных правил и шагов, за некоторыми исключениями. Единственная разница при решении линейных уравнений — это операция умножения или деления на отрицательное число.Умножение или деление неравенства на отрицательное число изменяет символ неравенства.

    Линейные неравенства можно решить с помощью следующих операций:

    • Сложение
    • Вычитание
    • Умножение
    • Деление
    • Распределение собственности

    Решение линейных неравенств с добавлением

    Давайте посмотрим на несколько примеров ниже, чтобы понять это понятие.

    Пример 1

    Решите 3x — 5 ≤ 3 — x.

    Решение

    Начнем с добавления обеих сторон неравенства на 5

    3x — 5 + 5 ≤ 3 + 5 — x

    3x ≤ 8 — x

    Затем сложим обе стороны на x.

    3x + x ≤ 8 — x + x

    4x ≤ 8

    Наконец, разделите обе части неравенства на 4, чтобы получить;

    x ≤ 2

    Пример 2

    Вычислите диапазон значений y, который удовлетворяет неравенству: y — 4 <2y + 5.

    Решение

    Сложите обе части неравенства на 4.

    y — 4 + 4 <2y + 5 + 4

    y <2y + 9

    Вычтите обе стороны на 2y.

    y — 2y <2y - 2y + 9

    Y <9 Умножьте обе части неравенства на -1 и измените направление символа неравенства. y> — 9

    Решение линейных неравенств с вычитанием

    Давайте рассмотрим несколько примеров ниже, чтобы понять эту концепцию.

    Пример 3

    Решите x + 8> 5.

    Решение

    Изолируйте переменную x, вычтя 8 из обеих сторон неравенства.

    x + 8-8> 5-8 => x> −3

    Следовательно, x> −3.

    Пример 4

    Решите 5x + 10> 3x + 24.

    Решение

    Вычтите 10 из обеих сторон неравенства.

    5x + 10-10> 3x + 24-10

    5x> 3x + 14.

    Теперь вычтем обе части неравенства на 3x.

    5x — 3x> 3x — 3x + 14

    2x> 14

    x> 7

    Решение линейных неравенств с умножением

    Давайте рассмотрим несколько примеров ниже, чтобы понять эту концепцию.

    Пример 5

    Решить x / 4> 5

    Решение:

    Умножить обе стороны неравенства на знаменатель дроби

    4 (x / 4)> 5 x 4

    x> 20

    Пример 6

    Решите -x / 4 ≥ 10

    Решение:

    Умножьте обе стороны неравенства на 4.

    4 (-x / 4) ≥ 10 x 4

    -x ≥ 40

    Умножьте обе стороны неравенства на -1 и измените направление символа неравенства на противоположное.

    x ≤ — 40

    Решение линейных неравенств с делением

    Давайте рассмотрим несколько примеров ниже, чтобы понять эту концепцию.

    Пример 7

    Решите неравенство: 8x — 2> 0.

    Решение

    Прежде всего, сложите обе стороны неравенства на 2

    8x — 2 + 2> 0 + 2

    8x> 2

    Теперь решите, разделив обе части неравенства на 8, чтобы получить;

    x> 2/8

    x> 1/4

    Пример 8

    Решите следующее неравенство:

    −5x> 100

    Решение

    Разделите оба сторон неравенства на -5 и измените направление символа неравенства

    = −5x / -5 <100 / -5

    = x <- 20

    Решение линейных неравенств с использованием свойства распределения

    Давайте посмотрим на несколько примеров ниже, чтобы понять эту концепцию.

    Пример 9

    Решить: 2 (x — 4) ≥ 3x — 5

    Решение

    2 (x — 4) ≥ 3x — 5

    Примените свойство распределения, чтобы удалить скобки.

    ⟹ 2x — 8 ≥ 3x — 5

    Сложить обе стороны на 8.

    ⟹ 2x — 8 + 8 ≥ 3x — 5 + 8

    ⟹ 2x ≥ 3x + 3

    Вычесть обе стороны на 3.

    ⟹ 2x — 3x ≥ 3x + 3 — 3x

    ⟹ -x ≥ 3

    ⟹ x ≤ — 3

    Пример 10

    Студент набрал 60 баллов за первый тест и 45 баллов во втором тесте заключительного экзамена.Сколько минимальных баллов должен набрать ученик в третьем тесте, получив в среднем не менее 62 баллов?

    Решение

    Пусть в третьем тесте будет набрано x баллов.

    (60 + 45 + x) / 3 ≥ 62
    105 + x ≥ 196
    x ≥ 93
    Следовательно, учащийся должен набрать 93 балла, чтобы поддерживать среднее значение не менее 62 баллов.

    Пример 11

    Джастину требуется не менее 500 долларов для празднования своего дня рождения.Если он уже накопил 150 долларов, до этой даты осталось 7 месяцев. Какую минимальную сумму он должен откладывать ежемесячно?

    Решение

    Пусть минимальная ежемесячная экономия = x

    150 + 7x ≥ 500

    Решить для x

    150-150 + 7x ≥ 500-150

    x ≥ 50

    Следовательно, Джастин должен экономить 50 долларов и более

    Пример 12

    Найдите два последовательных нечетных числа, которые больше 10 и имеют сумму меньше 40.

    Решение

    Пусть меньшее нечетное число = x

    Следовательно, следующее число будет x + 2

    x> 10 ………. больше 10

    x + (x + 2) <40 …… сумма меньше 40

    Решите уравнения.

    2x + 2 <40

    x + 1 <20

    x <19

    Объедините два выражения.

    10

    Следовательно, последовательные нечетные числа — 11 и 13, 13 и 15, 15 и 17, 17 и 19.

    Неравенства и числовая линия

    Лучшим инструментом для представления и визуализации чисел является числовая линия. Числовая линия определяется как прямая горизонтальная линия с числами, расположенными на равных отрезках или интервалах. У числовой прямой есть нейтральная точка в середине, известная как начало координат. Справа от начала координат на числовой прямой находятся положительные числа, а слева от начала координат — отрицательные числа.

    Линейные уравнения также можно решить графическим методом с использованием числовой прямой.Например, чтобы построить x> 1 на числовой прямой, вы обведите цифру 1 на числовой прямой и проведете линию, идущую от круга в направлении чисел, которые удовлетворяют утверждению о неравенстве.

    Пример 13

    Если символ неравенства больше или равен или меньше или равен знаку (≥ или ≤), нарисуйте круг над числовым числом и заполните или заштрихуйте круг.Наконец, проведите линию, идущую от заштрихованного круга в направлении чисел, которая удовлетворяет уравнению неравенства.

    Пример 14

    x ≥ 1

    Та же процедура используется для решения уравнений, включающих интервалы.

    Пример 15

    –2 < x <2

    Пример 16

    –1 ≤ x ≤ 2

    047

    Пример 17

    –1 < x ≤ 2

    Практические вопросы

    Решите следующие неравенства и представьте свой ответ на числовой прямой.

    1. 2x> 9
    2. x + 5> 13
    3. −3x <4
    4. 7x + 11> 2x + 5
    5. 2 (x + 3)
    6. — 5 ≤ 2x — 7 ≤ 1
    7. 4x — 8 ≤ 12

    Ответы

    1. x> 9/2
    2. x> 8
    3. x> −4/3
    4. x> −6/5
    5. x <−5.
    6. 1 ≤ x ≤ 4.
    7. x ≤ 5
    Предыдущий урок | Главная страница | Следующий урок

    Факты о неравенстве для детей

    Неравенство — это когда один объект равен:

    • Меньше другого (означает, что a меньше b )
    • Больше, чем другой (означает, что a больше, чем b )
    • Не меньше другого (означает, что a не меньше b , то есть либо больше, либо равно b )
    • Не больше другого (означает, что a не больше b , либо меньше или равно b )

    Неравенство иногда используется для обозначения утверждения, что одно выражение меньше, больше, не меньше или не больше другого.

    Работа с неравенствами

    Неравенство 1 Это решение уравнения x + 4> 12

    Неравенство в математике — это когда два решения или ответа сравниваются на большее или меньшее чем. Это когда сравниваются два или все же множество решений, не равных по количеству. Решить неравенство — значит найти его решения. Когда вы подставляете число в переменную, и утверждение истинно, тогда это решение. Когда вы подставляете число в переменную, и тогда утверждение неверно, число не является решением этого утверждения.

    Решение неравенства — это поиск решения данной переменной. Это поиск относительного порядка набора. Неравенство может иметь множество решений, но обычно мы ищем только решения в виде вещественных чисел. Правильный способ читать неравенство — слева направо, как и другие уравнения, но с той лишь разницей, что они имеют разные правила для каждого уравнения.

    Например, рассмотрим неравенство x + 4> 12, где x — действительное число. Во-первых, нужно найти x и убедиться, что это действительно решение.Ответ будет x> 8, и это верное утверждение. Это выражение касается положения x в наборе действительных чисел. Числовая линия — это один из способов показать местоположение относительно всех других действительных чисел (см. Рисунок Неравенство 1)

    Различные виды неравенства

    Линейное неравенство Пример линейного неравенства

    Существует пять различных видов неравенства:

    1. Первый вид — это линейные неравенства, которые представляют собой неравенства, различающие выражения на меньшее или равное, меньшее или большее или равное, большее чем.Он заключается в том, что если мы заменим неравенство отношением равенства, то результатом будет линейное уравнение.
    2. Второй вид — это комбинации неравенств, которые должны удовлетворять неравенствам. Здесь нужно сначала иметь число в наборах решений, так что числа, удовлетворяющие неравенствам, будут значениями в пересечении двух наборов решений.
    3. Третий вид — это неравенства, включающие абсолютные значения, что означает, что значения можно перефразировать как комбинации неравенств, включающих абсолютные значения.
    4. Четвертый тип неравенств называется полиномиальными неравенствами, что означает, что они непрерывны и их графики не имеют скачков или разрывов.
    5. И последнее, но не менее важное, это рациональные неравенства, что означает, что это форма одного полинома, деленного на другой многочлен. Другими словами, графики рациональных функций не имеют разрывов и не представлены в нулях знаменателя.
    абсолютное значение Пример, показывающий абсолютное значение

    Четыре способа решения неравенств

    Пример добавления неравенств.

    Есть четыре способа решить квадратные уравнения:

    1. Сложите или вычтите одинаковое число с обеих сторон.
    2. Сдвиньте стороны и измените расположение знака неравенства.
    3. Умножьте одинаковое число с обеих сторон.
    4. Разделите одинаковое положительное или отрицательное число на обе стороны. пример умножения неравенства

    Кроме того, для устранения неравенства потребуется два шага. Первый — упростить, используя обратное сложение или вычитание.Второй — еще больше упростить, используя обратную величину умножения или деления. Обратите внимание, что при умножении или делении неравенства на отрицательное число символ неравенства должен быть перевернут.

    Примеры решения неравенств

    Неравенство 2 Решение уравнения -6y

    Неравенство — это математическое утверждение, объясняющее, что эти два значения не равны и различны. Уравнение a b означает, что a не равно b .Неравенство одинаково с любым уравнением, с той (единственной) разницей, что неравенство не использует знак равенства, а символы неравенства. Неравенство b> a означает, что b больше a. Ограничения скорости, знаки и другие используют неравенство, чтобы выразить их.

    Решая неравенство, человек должен иметь верное утверждение. Когда вы делите или умножаете неравенство с отрицательным числом с обеих сторон, утверждение неверно. Чтобы утверждение было правильным с отрицательным числом, нужно изменить направление символа неравенства.Когда число является положительным числом, переворачивать символ не нужно. Неравенство — это правдивое заявление.

    Например, начните с истинного утверждения -6 y <-12. Когда обе стороны разделены на -6, результат станет y <2. В этом утверждении символ необходимо перевернуть, чтобы получить истинное утверждение, которое дает y> 2 как правильный ответ. В числовой строке (см. Рисунок Неравенство 2) замкнутый заштрихованный кружок указывает на то, что он включен в набор решений.Открытый кружок указывает на то, что он не входит в набор решений.

    Связанные страницы

    Решить неравенства | Начальная алгебра

    Результаты обучения

    • Опишите решения проблемы неравенства
      • Изобразите неравенства на числовой прямой
      • Изобразите неравенства, используя обозначение интервалов
    • Решите пошаговые неравенства
      • Используйте свойства сложения и умножения, чтобы решать алгебраические неравенства и выражать их решения графически и с интервальной нотацией
      • Решите неравенства, содержащие абсолютное значение
    • Решите многоступенчатые неравенства
      • Объединение свойств неравенства для выделения переменных, решения алгебраических неравенств и графического выражения их решений
      • Упростите и решите алгебраические неравенства, используя свойство распределенности для удаления скобок и дробей

    Изобразите неравенства на числовой прямой

    Во-первых, давайте определимся с важной терминологией.Неравенство — это математическое утверждение, которое сравнивает два выражения, используя идеи больше или меньше чем. В этих утверждениях используются специальные символы. Когда вы читаете неравенство, читайте его слева направо — как если бы вы читали текст на странице. В алгебре неравенства используются для описания больших наборов решений. Иногда существует бесконечное количество чисел, которые удовлетворяют неравенству, поэтому вместо того, чтобы пытаться перечислить бесконечное количество чисел, мы разработали несколько способов краткого описания очень больших списков.

    Первый способ, с которым вы, вероятно, знакомы — основное неравенство. Например:

    • [latex] {x} \ lt {9} [/ latex] указывает список чисел, которые меньше 9. Вы бы предпочли написать [latex] {x} \ lt {9} [/ latex] или попробовать перечислить все возможные числа меньше 9? (надеюсь, ваш ответ отрицательный)
    • [латекс] -5 \ le {t} [/ latex] указывает все числа, которые больше или равны [latex] -5 [/ latex].

    Обратите внимание, как размещение переменной слева или справа от знака неравенства может изменить, ищете ли вы больше или меньше.

    Например:

    • [латекс] x \ lt5 [/ latex] означает все действительные числа, которые меньше 5, тогда как;
    • [latex] 5 \ lt {x} [/ latex] означает, что 5 меньше x, или мы могли бы переписать это с x слева: [latex] x \ gt {5} [/ latex] обратите внимание, как неравенство по-прежнему указывает то же направление относительно x. Этот оператор представляет все действительные числа, которые больше 5, что легче интерпретировать, чем 5 меньше x.

    Второй способ — график с использованием числовой прямой:

    И третий способ — с интервалом.

    В этом разделе мы подробно рассмотрим второй и третий способы. Опять же, эти три способа написать решения неравенства:

    • неравенство
    • интервал
    • график

    Знаки неравенства

    В поле ниже показан символ, значение и пример для каждого знака неравенства. Иногда легко запутаться в неравенствах, просто не забывайте читать их слева направо.

    Символ слов Пример
    [латекс] \ neq [/ латекс] не равно [латекс] {2} \ neq {8} [/ latex], 2 равно не равно — 8 .
    [латекс] \ gt [/ латекс] больше [латекс] {5} \ gt {1} [/ latex], 5 больше, чем 1
    [латекс] \ lt [/ латекс] менее [латекс] {2} \ lt {11} [/ latex], 2 меньше 11
    [латекс] \ geq [/ латекс] больше или равно [латекс] {4} \ geq {4} [/ latex], 4 больше или равно 4
    [латекс] \ leq [/ латекс] меньше или равно [латекс] {7} \ leq {9} [/ latex], 7 меньше или равно 9

    Неравенство [латекс] x> y [/ latex] можно также записать как [latex] {y} <{x} [/ latex].Стороны любого неравенства можно поменять местами, если символ неравенства между ними также перевернут.

    Графическое изображение неравенства

    Неравенства также можно изобразить на числовой прямой. Ниже приведены три примера неравенств и их графики. Графики — очень полезный способ визуализировать информацию, особенно когда эта информация представляет собой бесконечный список чисел!

    [латекс] х \ leq -4 [/ латекс]. Это переводится во все действительные числа в числовой строке, которые меньше или равны 4.

    [латекс] {x} \ geq {-3} [/ латекс]. Это переводится во все действительные числа в числовой строке, которые больше или равны -3.

    Каждый из этих графиков начинается с круга — открытого или замкнутого (заштрихованного) круга. Эту точку часто называют конечной точкой решения. Замкнутый или заштрихованный круг используется для обозначения неравенств , которые больше или равны [latex] \ displaystyle \ left (\ geq \ right) [/ latex] или меньше или равно [latex] \ displaystyle. \ left (\ leq \ right) [/ латекс].Дело в том, что это часть решения. Открытый кружок используется для значений больше (>) или меньше (<). Дело в том, что , а не часть решения.

    Затем график бесконечно продолжается в одном направлении. Это показано линией со стрелкой в ​​конце. Например, обратите внимание, что для графа [latex] \ displaystyle x \ geq -3 [/ latex], показанного выше, конечной точкой является [latex] −3 [/ latex], представленная замкнутым кружком, поскольку неравенство равно больше или равно [латекс] -3 [/ латекс].Синяя линия рисуется справа от числовой, потому что значения в этой области больше, чем [latex] −3 [/ latex]. Стрелка в конце указывает, что решения продолжаются бесконечно.

    Пример

    График неравенства [латекс] x \ ge 4 [/ латекс]

    Показать решение

    Мы можем использовать числовую линию, как показано. Поскольку значения x включают 4, мы помещаем сплошную точку на числовой прямой с номером 4.

    Затем мы рисуем линию, которая начинается с [latex] x = 4 [/ latex] и, как указано стрелкой, продолжается до положительной бесконечности, что показывает, что набор решений включает все действительные числа, большие или равные 4.

    В этом видео показан пример построения графика неравенства.

    Пример

    Напишите неравенство, описывающее все действительные числа на числовой прямой, которые меньше 2, а затем нарисуйте соответствующий график.

    Показать решение

    Нам нужно начинать слева и работать вправо, поэтому мы начинаем с отрицательной бесконечности и заканчиваем на [latex] -2 [/ latex]. Мы не будем включать ни то, ни другое, потому что бесконечность не является числом, и неравенство не включает [латекс] -2 [/ латекс].

    Неравенство: [латекс] x <2 [/ латекс]

    Чтобы нарисовать график, сначала поместите открытую точку на числовой прямой, а затем нарисуйте линию, идущую влево. Нарисуйте стрелку в самой левой точке линии, чтобы указать, что она продолжается до бесконечности.

    В следующем видео показано, как математически написать неравенство, если оно выражается словами. Затем мы построим его график.

    Изобразите неравенства, используя обозначение интервалов

    Другой широко используемый и, возможно, самый краткий метод описания неравенств и решений неравенств называется интервальной нотацией . Согласно этому соглашению, наборы состоят из круглых или квадратных скобок, каждая из которых имеет свое значение. Решения для [latex] x \ geq 4 [/ latex] представлены как [latex] \ left [4, \ infty \ right) [/ latex]. Этот метод широко используется и будет присутствовать в других курсах математики, которые вы, возможно, пройдете.

    Основная концепция, которую следует запомнить, заключается в том, что круглые скобки представляют решения больше или меньше числа, а квадратные скобки представляют решения, которые больше или равны или меньше или равны числу.Используйте круглые скобки для обозначения бесконечности или отрицательной бесконечности, поскольку положительная и отрицательная бесконечность не являются числами в обычном смысле этого слова и, следовательно, не могут быть «равны». Несколько примеров интервала или набора чисел, в который попадает решение: [latex] \ left [-2,6 \ right) [/ latex] или все числа между [latex] -2 [/ латекс] и [латекс] 6 [/ латекс], включая [латекс] -2 [/ латекс], но не включая [латекс] 6 [/ латекс]; [latex] \ left (-1,0 \ right) [/ latex], все действительные числа между, но не включая [latex] -1 [/ latex] и [latex] 0 [/ latex]; и [latex] \ left (- \ infty, 1 \ right] [/ latex], все действительные числа меньше, чем [latex] 1 [/ latex] включительно.В таблице ниже представлены возможные варианты. Не забывайте читать неравенства слева направо, как текст.

    В таблице ниже описаны все возможные неравенства, которые могут возникнуть, и способы их записи с использованием интервальной записи, где a и b — действительные числа.

    Неравенство слов Интервальное обозначение
    [латекс] {a} \ lt {x} \ lt {b} [/ латекс] все действительные числа от a до b , кроме a и b [латекс] \ влево (а, б \ вправо) [/ латекс]
    [латекс] {x} \ gt {a} [/ латекс] Все действительные числа больше a , но не включая a [латекс] \ left (a, \ infty \ right) [/ latex]
    [латекс] {x} \ lt {b} [/ латекс] Все действительные числа меньше b , но не включая b [латекс] \ влево (- \ infty, b \ right) [/ латекс]
    [латекс] {x} \ ge {a} [/ латекс] Все действительные числа больше a , включая a [латекс] \ left [a, \ infty \ right) [/ latex]
    [латекс] {x} \ le {b} [/ latex] Все действительные числа меньше b , включая b [латекс] \ влево (- \ infty, b \ right] [/ латекс]
    [латекс] {a} \ le {x} \ lt {b} [/ latex] Все действительные числа между a и b , включая a [латекс] \ влево [а, б \ вправо) [/ латекс]
    [латекс] {a} \ lt {x} \ le {b} [/ latex] Все действительные числа от a до b , включая b [латекс] \ слева (a, b \ справа] [/ латекс]
    [латекс] {a} \ le {x} \ le {b} [/ latex] Все действительные числа от a до b , включая a и b [латекс] \ слева [a, b \ right] [/ латекс]
    [латекс] {x} \ lt {a} \ text {или} {x} \ gt {b} [/ latex] Все действительные числа меньше a или больше b [латекс] \ left (- \ infty, a \ right) \ чашка \ left (b, \ infty \ right) [/ latex]
    Все вещественные числа Все вещественные числа [латекс] \ влево (- \ infty, \ infty \ right) [/ латекс]

    Пример

    Опишите неравенство [латекс] x \ ge 4 [/ latex], используя обозначение интервала

    Показать решение

    Решения для [latex] x \ ge 4 [/ latex] представлены как [latex] \ left [4, \ infty \ right) [/ latex].

    Обратите внимание на использование кронштейна слева, потому что 4 включены в набор решений.

    В следующем видео мы показываем еще один пример использования обозначения интервалов для описания неравенства.

    Пример

    Используйте обозначение интервала, чтобы указать все действительные числа, большие или равные [latex] -2 [/ latex].

    Показать решение

    Используйте скобку слева от [latex] -2 [/ latex] и скобки после бесконечности: [latex] \ left [-2, \ infty \ right) [/ latex].Скобка указывает, что [latex] -2 [/ latex] включен в набор со всеми действительными числами от [latex] -2 [/ latex] до бесконечности.

    В следующем видео мы показываем еще один пример преобразования слов в неравенство и записи его в интервальной нотации, а также рисование графика.

    Подумай об этом

    В предыдущих примерах вам давали неравенство или описание одного со словами и просили нарисовать соответствующий график и записать интервал.В этом примере вам дается интервал и предлагается написать неравенство и нарисовать график.

    Дано [латекс] \ left (- \ infty, 10 \ right) [/ latex], запишите соответствующее неравенство и нарисуйте график.

    В поле ниже запишите, считаете ли вы, что сначала будет проще нарисовать график или сначала записать неравенство.

    Показать решение

    Сначала нарисуем график.

    Интервал читается как «все действительные числа меньше 10», поэтому мы начнем с того, что поставим точку на 10 и проведем линию влево со стрелкой, указывающей, что решение продолжается до отрицательной бесконечности.

    Чтобы записать неравенство, мы будем использовать <, поскольку круглые скобки указывают, что 10 не включено. [латекс] x <10 [/ латекс]

    В следующем видео вы увидите примеры того, как нарисовать график с учетом неравенства в обозначении интервалов.

    И, наконец, последнее видео, в котором показано, как записывать неравенства с помощью графика, с интервальной нотацией и в виде неравенства.

    Решите пошаговые неравенства

    Решите неравенства сложением и вычитанием

    Вы можете решить большинство неравенств, используя обратные операции, как вы это делали для решения уравнений.Это потому, что, когда вы добавляете или вычитаете одно и то же значение с обеих сторон неравенства, вы сохраняете неравенство. Эти свойства указаны в поле ниже.

    Сложение и вычитание свойств неравенства

    Если [латекс] a> b [/ латекс], , то [латекс] a + c> b + c [/ латекс].

    Если [латекс] a> b [/ latex] , , то [латекс] a-c> b-c [/ latex].

    Поскольку неравенство имеет несколько возможных решений, графическое представление решений дает полезную визуализацию ситуации, как мы видели в последнем разделе.В приведенном ниже примере показаны шаги для решения и графического представления неравенства, а также для выражения решения с использованием интервальной записи.

    Пример

    Найдите x.

    [латекс] {x} +3 \ lt {5} [/ латекс]

    Показать решение

    Полезно думать об этом неравенстве как о том, что вам предлагается найти все значения для x , включая отрицательные числа, так что при сложении трех вы получите число меньше 5.

    [латекс] \ displaystyle \ begin {array} {l} x + 3 <\, \, \, \, 5 \\\ подчеркивание {\, \, \, \, \, - 3 \, \, \, \, - 3} \\ x \, \, \, \, \, \, \, \, <\, \, \, \, 2 \, \, \ end {array} [/ latex]

    Выделите переменную, вычтя 3 из обеих частей неравенства.

    Ответ

    Неравенство: [латекс] x <2 [/ латекс]

    Интервал: [латекс] \ влево (- \ infty, 2 \ вправо) [/ латекс]

    График:

    Линия представляет всех чисел, к которым можно сложить 3 и получить число меньше 5. Есть много чисел, которые решают это неравенство!

    Так же, как вы можете проверить решение уравнения, вы можете проверить решение неравенства. Сначала вы проверяете конечную точку, подставляя ее в соответствующее уравнение.Затем вы проверяете, правильно ли неравенство, подставляя любое другое решение, чтобы увидеть, является ли оно одним из решений. Поскольку существует несколько решений, рекомендуется проверить более одного из возможных решений. Это также может помочь вам проверить правильность вашего графика.

    Пример ниже показывает, как вы можете проверить, что [latex] x <2 [/ latex] является решением для [latex] x + 3 <5 [/ latex] .

    Пример

    Убедитесь, что [latex] x <2 [/ latex] является решением для [latex] x + 3 <5 [/ latex].

    Показать решение

    Подставьте конечную точку 2 в соответствующее уравнение [латекс] x + 3 = 5 [/ latex].

    [латекс] \ begin {array} {r} x + 3 = 5 \\ 2 + 3 = 5 \\ 5 = 5 \ end {array} [/ latex]

    Выберите значение меньше 2, например 0, чтобы проверить неравенство. (Это значение будет на затененной части графика.)

    [латекс] \ displaystyle \ begin {array} {r} x + 3 <5 \\ 0 + 3 <5 \\ 3 <5 \ end {array} [/ latex]

    Проверяет!

    [латекс] x <2 [/ латекс] - это решение [латекс] x + 3 <5 [/ латекс].

    В следующих примерах показаны проблемы неравенства, которые включают операции с отрицательными числами. Также показан график решения неравенства. Не забудьте проверить решение. Это хорошая привычка!

    Пример

    Решите для x : [латекс] x-10 \ leq-12 [/ латекс]

    Показать решение

    Изолируйте переменную, добавив 10 к обеим сторонам неравенства.

    [латекс] \ Displaystyle \ begin {array} {r} x-10 \ le -12 \\\ подчеркивание {\, \, \, + 10 \, \, \, \, \, + 10} \\ x \, \, \, \, \, \, \, \, \, \, \ le \, \, \, — 2 \ end {array} [/ latex]

    Ответ

    Неравенство: [латекс] x \ leq-2 [/ latex]
    Интервал: [latex] \ left (- \ infty, -2 \ right] [/ latex]
    График: Обратите внимание, что используется замкнутый круг, потому что неравенство «меньше или равно» [латекс] \ left (\ leq \ right) [/ latex].Синяя стрелка нарисована слева от точки [латекс] -2 [/ латекс], потому что это значения меньше, чем [латекс] -2 [/ латекс].

    Проверьте раствор [латекс] x-10 \ leq -12 [/ latex]

    Показать решение

    Подставьте конечную точку [латекс] −2 [/ latex] в соответствующее уравнение [латекс] x-10 = −12 [/ latex]

    [латекс] \ displaystyle \ begin {array} {r} x-10 = -12 \, \, \, \\\ text {Does} \, \, \, — 2-10 = -12? \\ — 12 = -12 \, \, \, \ end {array} [/ latex]

    Выберите значение меньше [латекс] -2 [/ латекс], например [латекс] -5 [/ латекс], чтобы проверить неравенство.(Это значение будет на затененной части графика.)

    [латекс] \ displaystyle \ begin {array} {r} x-10 \ le -12 \, \, \, \\\ text {} \, \ text {Is} \, \, — 5-10 \ le -12? \\ — 15 \ le -12 \, \, \, \\\ text {It} \, \ text {проверяет!} \ End {array} [/ latex]

    [латекс] x \ leq -2 [/ latex] — это решение [латекса] x-10 \ leq -12 [/ latex]

    Пример

    Решите относительно a . [латекс] a-17> -17 [/ латекс]

    Показать решение

    Изолируйте переменную, прибавив 17 к обеим сторонам неравенства.

    [латекс] \ displaystyle \ begin {array} {r} a-17> -17 \\\ подчеркивание {\, \, \, + 17 \, \, \, \, \, + 17} \\ a \ , \, \, \, \, \, \, \, \, \, \,> \, \, \, \, \, \, 0 \ end {array} [/ latex]

    Ответ

    Неравенство: [латекс] \ displaystyle a \, \,> \, 0 [/ latex]

    Интервал: [latex] \ left (0, \ infty \ right) [/ latex] Обратите внимание, как мы используем скобки слева, чтобы показать, что решение не включает 0.

    График

    : обратите внимание на пустой кружок, чтобы показать, что решение не включает 0.

    Проверьте раствор [латекс] a-17> -17 [/ latex]

    Показать решение

    [latex] \ displaystyle a \, \,> \, 0 [/ latex] — правильное решение для [latex] a-17> -17 [/ latex]?

    Подставьте конечную точку 0 в соответствующее уравнение.

    [латекс] \ displaystyle \ begin {array} {r} a-17 = -17 \, \, \, \\\ text {Does} \, \, \, 0-17 = -17? \\ — 17 = -17 \, \, \, \ end {array} [/ latex]

    Выберите значение больше 0, например 20, чтобы проверить неравенство. (Это значение будет на затененной части графика.)

    [латекс] \ displaystyle \ begin {array} {r} a-17> -17 \, \, \, \\\ text {Is} \, \, 20-17> -17? \\ 3> -17 \, \, \, \\\\\ text {Проверяет!} \, \, \, \, \ end {array} [/ latex]

    [latex] \ displaystyle a \,> \, 0 [/ latex] — это решение для [latex] a-17> -17 [/ latex]

    Предыдущие примеры показали вам, как решить одношаговое неравенство с переменной в левой части.В следующем видео представлены примеры того, как разрешить однотипное неравенство.

    Что бы вы сделали, если бы переменная находилась в правой части неравенства? В следующем примере вы увидите, как справиться с этим сценарием.

    Пример

    Решите для x : [латекс] 4 \ geq {x} +5 [/ латекс]

    Показать решение

    Изолируйте переменную, добавив 10 к обеим сторонам неравенства.

    [латекс] \ displaystyle \ begin {array} {r} 4 \ geq {x} +5 \\\ underline {\, \, \, — 5 \, \, \, \, \, — 5} \\ -1 \, \, \, \, \, \, \, \, \, \, \ ge \, \, \, x \ end {array} [/ latex]

    Перепишите неравенство с переменной слева — это упростит запись интервала и построение графика.

    [латекс] x \ le {-1} [/ латекс]

    Обратите внимание, как острая часть неравенства по-прежнему направлена ​​на переменную, поэтому вместо того, чтобы читать, как отрицательное значение больше или равно x, теперь оно читается как x меньше или равно отрицательному.

    Ответ

    Неравенство: [latex] x \ le {-1} [/ latex] Это также можно записать как
    Interval: [latex] \ left (- \ infty, -1 \ right] [/ latex]
    График: Обратите внимание, что замкнутый круг используется, потому что неравенство «меньше или равно».Синяя стрелка нарисована слева от точки [latex] -1 [/ latex], потому что это значения меньше, чем [latex] -1 [/ latex].

    Проверьте решение для [latex] 4 \ geq {x} +5 [/ latex]

    Показать решение

    Подставьте конечную точку [латекс] -1 [/ латекс] в соответствующее уравнение [латекс] 4 = x + 5 [/ latex]

    [латекс] \ displaystyle \ begin {array} {r} 4 = x + 5 \, \, \, \\\ text {Does} \, \, \, 4 = -1 + 5? \\ — 1 = -1 \, \, \, \ end {array} [/ latex]

    Выберите значение меньше [латекс] -1 [/ латекс], например [латекс] -5 [/ латекс], чтобы проверить неравенство.(Это значение будет на затененной части графика.)

    [латекс] \ displaystyle \ begin {array} {r} 4 \ geq {-5} +5 \, \, \, \\\ text {} \, \ text {Is} \, \, 4 \ ge 0 ? \\\ text {It} \, \ text {проверяет!} \ end {array} [/ latex]

    [latex] x \ le {-1} [/ latex] — это решение [latex] 4 \ geq {x} +5 [/ latex]

    В следующем видео показаны примеры решения неравенств с переменной справа.

    Решите неравенства умножением и делением

    Решение неравенства с переменной с коэффициентом, отличным от 1, обычно включает умножение или деление.Эти шаги подобны решению одношаговых уравнений, включающих умножение или деление, ЗА ИСКЛЮЧЕНИЕМ знака неравенства. Давайте посмотрим, что происходит с неравенством, когда вы умножаете или делите каждую сторону на одно и то же число.

    Начнем с истинного утверждения:

    [латекс] 10> 5 [/ латекс]

    Давайте попробуем еще раз, начав с того же истинного утверждения:

    [латекс] 10> 5 [/ латекс]

    Затем умножьте обе стороны на одинаковое положительное число:

    [латекс] 10 \ cdot 2> 5 \ cdot 2 [/ латекс]

    На этот раз умножьте обе стороны на одинаковое отрицательное число:

    [латекс] 10 \ cdot-2> 5 \\ \, \, \, \, \, \ cdot -2 \, \ cdot-2 [/ латекс]

    20 больше 10, поэтому у вас все еще есть истинное неравенство:

    [латекс] 20> 10 [/ латекс]

    Погодите! [latex] −20 [/ latex] — это , а не больше, чем [latex] −10 [/ latex], поэтому ваше утверждение неверно.

    [латекс] -20> -10 [/ латекс]

    При умножении на положительное число оставьте знак неравенства как есть! Вы должны «перевернуть» знак неравенства, чтобы утверждение стало верным:

    [латекс] -20 <-10 [/ латекс]

    Внимание! Когда вы умножаете или делите на отрицательное число, «переверните» знак неравенства. Всякий раз, когда вы умножаете или делите обе стороны неравенства на отрицательное число, знак неравенства должен быть перевернут, чтобы утверждение оставалось верным.Эти правила кратко изложены во вставке ниже.


    Свойства неравенства умножения и деления

    Начать с Умножить на Окончательное неравенство
    [латекс] a> b [/ латекс] [латекс] c [/ латекс] [латекс] ac> bc [/ латекс]
    [латекс] a> b [/ латекс] [латекс] -c [/ латекс] [латекс] ac
    Начать с Разделить на Окончательное неравенство
    [латекс] a> b [/ латекс] [латекс] c [/ латекс] [латекс] \ displaystyle \ frac {a} {c}> \ frac {b} {c} [/ latex]
    [латекс] a> b [/ латекс] [латекс] -c [/ латекс] [латекс] \ displaystyle \ frac {a} {c} <\ frac {b} {c} [/ latex]

    Имейте в виду, что вы меняете знак только при умножении и делении на отрицательное число .Если вы добавите или вычтете на отрицательное число, неравенство останется прежним.

    Пример

    Найдите x. [латекс] 3x> 12 [/ латекс]

    Показать решение Разделите обе части на 3, чтобы изолировать переменную.

    [латекс] \ displaystyle \ begin {array} {r} \ underline {3x}> \ underline {12} \\ 3 \, \, \, \, \, \, \, \, \, \, \, \, 3 \\ x> 4 \, \, \, \ end {array} [/ latex]

    Проверьте свое решение, сначала проверив конечную точку 4, а затем проверив другое решение на предмет неравенства.

    [латекс] \ begin {array} {r} 3 \ cdot4 = 12 \\ 12 = 12 \\ 3 \ cdot10> 12 \\ 30> 12 \\\ text {Проверяет!} \ End {array} [/ латекс]

    Ответ

    Неравенство: [латекс] \ displaystyle x> 4 [/ latex]

    Интервал: [латекс] \ left (4, \ infty \ right) [/ latex]

    График:

    Не было необходимости вносить какие-либо изменения в знак неравенства, потому что обе части неравенства были разделены на положительных 3. В следующем примере есть деление на отрицательное число, поэтому есть дополнительный шаг в решении!

    Пример

    Решите для x .[латекс] -2x> 6 [/ латекс]

    Показать решение Разделите каждую сторону неравенства на [latex] −2 [/ latex], чтобы изолировать переменную, и измените направление знака неравенства из-за деления на отрицательное число.

    [латекс] \ displaystyle \ begin {array} {r} \ underline {-2x} <\ underline {\, 6 \,} \\ - 2 \, \, \, \, - 2 \, \\ x < -3 \ end {массив} [/ латекс]

    Проверьте свое решение, сначала проверив конечную точку [latex] −3 [/ latex], а затем проверив другое решение на предмет неравенства.

    [латекс] \ begin {array} {r} -2 \ left (-3 \ right) = 6 \\ 6 = 6 \ -2 \ left (-6 \ right)> 6 \\ 12> 6 \ end {array} [/ latex]

    Проверяет!

    Ответ

    Неравенство: [латекс] \ displaystyle x <-3 [/ latex]

    Интервал: [латекс] \ left (- \ infty, -3 \ right) [/ latex]

    График:
    Поскольку обе стороны неравенства были разделены отрицательным числом [латекс] -2 [/ latex], символ неравенства был изменен с> на <.

    В следующем видео показаны примеры решения одношаговых неравенств с использованием свойства равенства умножения, где переменная находится в левой части.

    Подумай об этом

    Прежде чем читать решение следующего примера, подумайте, какие свойства неравенств вам, возможно, потребуется использовать для решения неравенства. Чем этот пример отличается от предыдущего? Напишите свои идеи в поле ниже.

    Решите для x .[латекс] — \ frac {1} {2}> — 12x [/ латекс]

    Показать решение

    Это неравенство имеет переменную в правой части, которая отличается от предыдущих примеров. Начните процесс решения, как и раньше, а в конце вы можете переместить переменную влево, чтобы записать окончательное решение.

    Разделите обе стороны на [латекс] -12 [/ латекс], чтобы изолировать переменную. Поскольку вы делите на отрицательное число, вам нужно изменить направление знака неравенства.

    [латекс] \ displaystyle \ begin {array} {l} — \ frac {1} {2} \ gt {-12x} \\\\\ frac {- \ frac {1} {2}} {- 12} \ gt \ frac {-12x} {- 12} \\\ end {array} [/ latex]

    Для деления дроби на целое число необходимо умножить на обратную величину, а обратная величина [latex] -12 [/ latex] будет [latex] \ frac {1} {- 12} [/ latex]

    [латекс] \ displaystyle \ begin {array} {r} \ left (- \ frac {1} {12} \ right) \ left (- \ frac {1} {2} \ right) \ lt \ frac {- 12x} {- 12} \, \, \\\\ \ frac {1} {24} \ lt \ frac {\ cancel {-12} x} {\ cancel {-12}} \\\\ \ frac { 1} {24} \ lt {x} \, \, \, \, \, \, \, \, \, \, \ end {array} [/ latex]

    Ответ

    Неравенство: [latex] \ frac {1} {24} \ lt {x} [/ latex] Это также можно записать с переменной слева как [latex] x \ gt \ frac {1} {24} [ /латекс].Чтобы записать неравенство с переменной слева, нужно немного подумать, но это поможет вам записать интервал и правильно нарисовать график.

    Интервал: [латекс] \ left (\ frac {1} {24}, \ infty \ right) [/ latex]

    График:

    В следующем видео приведены примеры того, как решить неравенство со свойством умножения равенства, где переменная находится справа.

    Объединение свойств неравенства для решения алгебраических неравенств

    Популярная стратегия решения уравнений с выделением переменной также применима к решению неравенств.Путем сложения, вычитания, умножения и / или деления вы можете переписать неравенство так, чтобы переменная находилась с одной стороны, а все остальное — с другой. Как и в случае одношаговых неравенств, решения многоступенчатых неравенств можно изобразить на числовой прямой.

    Пример

    Решите относительно p . [латекс] 4p + 5 <29 [/ латекс]

    Показать решение

    Начните изолировать переменную, вычтя 5 из обеих частей неравенства.

    [латекс] \ displaystyle \ begin {array} {l} 4p + 5 <\, \, \, 29 \\\ подчеркивание {\, \, \, \, \, \, \, \, \, - 5 \, \, \, \, \, - 5} \\ 4p \, \, \, \, \, \, \, \, \, <\, \, 24 \, \, \ end {array} [ / латекс]

    Разделите обе части неравенства на 4, чтобы выразить переменную с коэффициентом 1.

    [латекс] \ begin {array} {l} \ underline {4p} \, <\, \, \ underline {24} \, \, \\\, 4 \, \, \, \, <\, \ , 4 \\\, \, \, \, \, p <6 \ end {array} [/ latex]

    Ответ

    Неравенство: [латекс] p <6 [/ латекс]

    Интервал: [латекс] \ влево (- \ infty, 6 \ вправо) [/ латекс]

    График

    : обратите внимание на белый кружок в конечной точке 6, чтобы показать, что решения неравенства не включают 6. Значения, где p меньше 6, находятся вдоль числовой прямой слева от 6.

    Проверьте решение.

    Показать решение

    Проверьте конечную точку 6 в соответствующем уравнении.

    [латекс] \ displaystyle \ begin {array} {r} 4p + 5 = 29 \, \, \, \\\ text {Does} \, \, \, 4 (6) + 5 = 29? \\ 24 + 5 = 29 \, \, \, \\ 29 = 29 \, \, \, \\\ текст {Да!} \, \, \, \, \, \, \ End {array} [/ latex]

    Попробуйте другое значение, чтобы проверить неравенство. Давайте использовать [latex] p = 0 [/ latex].

    [латекс] \ displaystyle \ begin {array} {r} 4p + 5 <29 \, \, \, \\\ text {Is} \, \, \, 4 (0) +5 <29? \\ 0 +5 <29 \, \, \, \\ 5 <29 \, \, \, \\\ text {Да!} \, \, \, \, \, \ End {array} [/ latex]

    [латекс] p <6 [/ latex] - это решение [латекс] 4p + 5 <29 [/ latex]

    Пример

    Решите для x : [латекс] 3x – 7 \ ge 41 [/ латекс]

    Показать решение

    Начните изолировать переменную, прибавив 7 к обеим сторонам неравенства, затем разделите обе стороны неравенства на 3, чтобы выразить переменную с коэффициентом 1.

    [латекс] \ displaystyle \ begin {array} {l} 3x-7 \ ge 41 \\\ подчеркивание {\, \, \, \, \, \, \, + 7 \, \, \, \, + 7} \\\ frac {3x} {3} \, \, \, \, \, \, \, \, \ ge \ frac {48} {3} \\\, \, \, \, \, \, \, \, \, \, x \ ge 16 \ end {array} [/ latex]

    Ответ

    Неравенство: [латекс] x \ ge 16 [/ латекс]

    Интервал: [латекс] \ left [16, \ infty \ right) [/ latex]

    График: Чтобы изобразить это неравенство, вы рисуете замкнутый круг в конечной точке 16 числовой линии, чтобы показать, что решения включают значение 16. Линия продолжается вправо от 16, потому что все числа больше 16 также будут составлять неравенство [латекс] 3x – 7 \ ge 41 [/ латекс] верно.

    Проверьте решение.

    Показать решение

    Сначала проверьте конечную точку 16 в соответствующем уравнении.

    [латекс] \ displaystyle \ begin {array} {r} 3x-7 = 41 \, \, \, \\\ text {Does} \, \, \, 3 (16) -7 = 41? \\ 48 -7 = 41 \, \, \, \\ 41 = 41 \, \, \, \\\ текст {Да!} \, \, \, \, \, \ End {array} [/ latex]

    Затем попробуйте другое значение, чтобы проверить неравенство. Давайте использовать [latex] x = 20 [/ latex].

    [латекс] \ displaystyle \ begin {array} {r} \, \, \, \, 3x-7 \ ge 41 \, \, \, \\\ текст {Is} \, \, \, \, \ , 3 (20) -7 \ ge 41? \\ 60-7 \ ge 41 \, \, \, \\ 53 \ ge 41 \, \, \, \\\ текст {Да!} \, \, \ , \, \, \ end {array} [/ latex]

    При решении многоступенчатых уравнений обращайте внимание на ситуации, в которых вы умножаете или делите на отрицательное число.В этих случаях необходимо перевернуть знак неравенства.

    Пример

    Решите относительно p . [латекс] −58> 14−6p [/ латекс]

    Показать решение

    Обратите внимание, как переменная находится в правой части неравенства, метод решения в этом случае не меняется.

    Начните изолировать переменную, вычтя 14 из обеих частей неравенства.

    [латекс] \ displaystyle \ begin {array} {l} −58 \, \,> 14−6p \\\ подчеркивание {\, \, \, \, \, \, \, \, \, \, \ , \, — 14 \, \, \, \, \, \, \, — 14} \\ — 72 \, \, \, \, \, \, \, \, \, \, \,> — 6p \ end {array} [/ latex]

    Разделите обе части неравенства на [latex] −6 [/ latex], чтобы выразить переменную с коэффициентом 1.При делении на отрицательное число знак неравенства меняется на противоположный.

    [латекс] \ begin {array} {l} \ underline {-72}> \ underline {-6p} \\ — 6 \, \, \, \, \, \, \, \, \, \, — 6 \\\, \, \, \, \, \, 12 \ lt {p} \ end {array} [/ latex]

    Мы также можем записать это как [latex] p> 12 [/ latex]. Обратите внимание, как знак неравенства все еще открывается в сторону переменной p.

    Ответ

    Неравенство: [латекс] p> 12 [/ latex]
    Интервал: [latex] \ left (12, \ infty \ right) [/ latex]
    График: График неравенства p > 12 имеет открытый кружок на 12 со стрелкой, уходящей вправо.

    Проверьте решение.

    Показать решение

    Сначала проверьте конечную точку 12 в соответствующем уравнении.

    [латекс] \ begin {array} {r} -58 = 14-6p \\ — 58 = 14-6 \ left (12 \ right) \\ — 58 = 14-72 \\ — 58 = -58 \ end {array} [/ latex]

    Затем попробуйте другое значение, чтобы проверить неравенство. Попробуйте 100.

    [латекс] \ begin {array} {r} -58> 14-6p \\ — 58> 14-6 \ left (100 \ right) \\ — 58> 14-600 \\ — 58> -586 \ end {array} [/ latex]

    В следующем видео вы увидите пример решения линейного неравенства с переменной в левой части неравенства и пример переключения направления неравенства после деления на отрицательное число.

    В следующем видео вы увидите пример решения линейного неравенства с переменной в правой части неравенства и пример переключения направления неравенства после деления на отрицательное число.

    Упростите и решите алгебраические неравенства, используя свойство распределенности

    Как и в случае с уравнениями, свойство распределения может применяться для упрощения выражений, являющихся частью неравенства.Как только скобки будут убраны, устранение неравенства будет несложным.

    Пример

    Решите для x . [латекс] 2 \ влево (3x – 5 \ вправо) \ leq 4x + 6 [/ латекс]

    Показать решение

    Раздать, чтобы убрать скобки.

    [латекс] \ displaystyle \ begin {array} {r} \, 2 (3x-5) \ leq 4x + 6 \\\, \, \, \, 6x-10 \ leq 4x + 6 \ end {array} [/ латекс]

    Вычтите 4 x с обеих сторон, чтобы получить переменный член только с одной стороны.

    [латекс] \ begin {array} {r} 6x-10 \ le 4x + 6 \\\ подчеркивание {-4x \, \, \, \, \, \, \, \, \, \, \, \ , \, \, — 4x} \, \, \, \, \, \, \, \, \, \\\, \, \, 2x-10 \, \, \ leq \, \, \, \ , \, \, \, \, \, \, \, \, 6 \ end {array} [/ latex]

    Добавьте 10 к обеим сторонам, чтобы изолировать переменную.

    [латекс] \ begin {array} {r} \\\, \, \, 2x-10 \, \, \ le \, \, \, \, \, \, \, \, 6 \, \, \, \\\ подчеркивание {\, \, \, \, \, \, + 10 \, \, \, \, \, \, \, \, \, + 10} \\\, \, \, 2x \, \, \, \, \, \, \, \, \, \, \, \ le \, \, \, \, \, 16 \, \, \, \ end {array} [/ latex ]

    Разделите обе части на 2, чтобы выразить переменную с коэффициентом 1.

    [латекс] \ begin {array} {r} \ underline {2x} \ le \, \, \, \ underline {16} \\\, \, \, 2 \, \, \, \, \, \ , \, \, \, \, \, \, \, 2 \, \, \\\, \, \, \, \, \, \, \, \, \, \, \, \, \, \, x \, \, \, \ le \, \, \, \, \, 8 \ end {array} [/ latex]

    Ответ

    Неравенство: [latex] x \ le8 [/ latex]
    Интервал: [latex] \ left (- \ infty, 8 \ right] [/ latex]
    График: График этого набора решений включает 8 и все, что слева от 8 в числовой строке.

    Проверьте решение.

    Показать решение

    Сначала проверьте конечную точку 8 в соответствующем уравнении.

    [латекс] \ displaystyle \ begin {array} {r} 2 (3x-5) = 4x + 6 \, \, \, \, \, \, \\ 2 (3 \, \ cdot \, 8-5 ) = 4 \, \ cdot \, 8 + 6 \\\, \, \, \, \, \, \, \, \, \, \, 2 (24-5) = 32 + 6 \, \, \, \, \, \, \\ 2 (19) = 38 \, \, \, \, \, \, \, \, \, \, \, \, \, \, \, \, \\ 38 = 38 \, \, \, \, \, \, \, \, \, \, \, \, \, \, \, \, \ end {array} [/ latex]

    Затем выберите другое решение и оцените неравенство для этого значения, чтобы убедиться, что это истинное утверждение.Попробуйте 0.

    [латекс] \ displaystyle \ begin {array} {l} 2 (3 \, \ cdot \, 0-5) \ le 4 \, \ cdot \, 0 + 6? \\\, \, \, \, \, \, \, \, \, \, \, \, \, \, \, 2 (-5) \ le 6 \\\, \, \, \, \, \, \, \, \, \, \, \, \, \, \, \, \, — 10 \ le 6 \, \, \ end {array} [/ latex]

    [latex] x \ le8 [/ latex] — это решение для [latex] \ left (- \ infty, 8 \ right] [/ latex]

    В следующем видео вам дается пример того, как решить многоступенчатое неравенство, которое требует использования свойства распределения.

    Подумай об этом

    В следующем примере вам дано неравенство с термином, который выглядит сложным.Если вы сделаете паузу и подумаете о том, как использовать порядок операций для устранения неравенства, мы надеемся, что это покажется простой проблемой. Используйте текстовое поле, чтобы записать, что вы считаете лучшим первым шагом.

    Решите для a. [латекс] \ displaystyle \ frac {{2} {a} — {4}} {{6}} {<2} [/ latex]

    Показать решение

    Очистите дробь, умножив обе части уравнения на 6.

    [латекс] \ displaystyle \ begin {array} {r} \ frac {{2} {a} — {4}} {{6}} {<2} \, \, \, \, \, \, \ , \, \\\\ 6 \, \ cdot \, \ frac {2a-4} {6} <2 \, \ cdot \, 6 \\\\ {2a-4} <12 \, \, \, \, \, \, \ end {array} [/ latex]

    Добавьте 4 к обеим сторонам, чтобы изолировать переменную.

    [латекс] \ displaystyle \ begin {array} {r} 2a-4 <12 \\\ underline {\, \, \, + 4 \, \, \, \, + 4} \\ 2a <16 \ end {array} [/ latex]

    Разделите обе части на 2, чтобы выразить переменную с коэффициентом 1.

    [латекс] \ displaystyle \ begin {array} {c} \ frac {2a} {2} <\, \ frac {16} {2} \\\\ a <8 \ end {array} [/ latex]

    Ответ

    Неравенство: [латекс] a <8 [/ латекс]

    Интервал: [латекс] \ влево (- \ infty, 8 \ вправо) [/ латекс]

    График: График этого решения содержит сплошную точку у 8, чтобы показать, что 8 входит в набор решений.Линия продолжается влево, чтобы показать, что значения меньше 8 также включены в набор решений.

    Проверьте решение.

    Показать решение Сначала проверьте конечную точку 8 в соответствующем уравнении.

    [латекс] \ displaystyle \ begin {array} {r} \ frac {2a-4} {6} = 2 \, \, \, \, \\\\\ text {Does} \, \, \, \ frac {2 (8) -4} {6} = 2? \\\\\ frac {16-4} {6} = 2 \, \, \, \, \\\\\ frac {12} {6 } = 2 \, \, \, \, \\\\ 2 = 2 \, \, \, \, \\\\\ text {Да!} \, \, \, \, \, \ End {массив } [/ латекс]

    Затем выберите другое решение и оцените неравенство для этого значения, чтобы убедиться, что это истинное утверждение.Попробуйте 5.

    [латекс] \ displaystyle \ begin {array} {r} \ text {Is} \, \, \, \ frac {2 (5) -4} {6} <2? \\\\\ frac {10- 4} {6} <2 \, \, \, \\\\\, \, \, \, \ frac {6} {6} <2 \, \, \, \\\\ 1 <2 \, \, \, \\\\\ text {Да!} \, \, \, \, \, \ end {array} [/ latex]

    Сводка

    Решение неравенств очень похоже на решение уравнений, за исключением того, что вам нужно перевернуть символы неравенства, когда вы умножаете или делите обе стороны неравенства на отрицательное число. Решения неравенств можно представить тремя способами: интервал, график и неравенство.Поскольку обычно существует более одного решения неравенства, когда вы проверяете свой ответ, вы должны проверить конечную точку и одно другое значение, чтобы проверить направление неравенства.

    На неравенства могут быть разные ответы. Решения часто отображаются в виде числовой линии, чтобы наглядно представить все решения. Многоступенчатые неравенства решаются с использованием тех же процессов, что и для решения уравнений, за одним исключением. Когда вы умножаете или делите обе стороны неравенства на отрицательное число, вы должны перевернуть символ неравенства.Символы неравенства остаются неизменными при добавлении или вычитании положительных или отрицательных чисел к обеим сторонам неравенства.

    Решение двухшаговых линейных неравенств

    Чтобы решить двухэтапное неравенство, сначала отмените сложение или вычитание, используя обратные операции , а затем отмените умножение или деление.

    Обратной операцией сложения является вычитание и наоборот.

    Точно так же обратная операция умножения — это деление, и наоборот.

    Обратите внимание, что всякий раз, когда вы умножаете или делите обе стороны неравенства на отрицательное число, обращайте неравенство.

    Пример 1:

    Решать 2 Икс + 1 < 7 .

    Во-первых, нам нужно изолировать переменный член на одной стороне неравенства. Здесь слева 1 добавляется к члену переменной, 2 Икс .Обратной операцией сложения является вычитание. Итак, вычтите 1 с обеих сторон.

    2 Икс + 1 — 1 < 7 - 1 2 Икс < 6

    Теперь у нас есть переменная Икс умножается на 2 .Обратная операция умножения — это деление. Итак, разделите обе стороны на 2 .

    2 Икс 2 < 6 2 Икс < 3

    То есть неравенство справедливо для всех значений Икс которые меньше чем 3 .

    Следовательно, решения неравенства 2 Икс + 1 < 7 все числа меньше чем 3 .

    Пример 2:

    Решать — 3 Икс — 8 ≥ — 2 .

    Сначала нам нужно выделить переменный член — 3 Икс налево. Обратной операцией вычитания является сложение. Итак, добавляем 8 в обе стороны.

    — 3 Икс — 8 + 8 ≥ — 2 + 8 — 3 Икс ≥ 6

    Чтобы изолировать переменную Икс , разделите обе стороны на — 3 .

    Обратите внимание, что всякий раз, когда вы умножаете или делите обе стороны неравенства на отрицательное число, обращайте неравенство.

    — 3 Икс — 3 ≤ 6 — 3 Икс ≤ — 2

    Следовательно, решения неравенства — 3 Икс — 8 ≥ — 2 все числа меньше или равны — 2 .

    1. Свойства неравенств

    Выражение `a

    a меньше b

    , а выражение `a> b` читается как

    a больше b.

    Знаки `<` и `>` определяют то, что известно как смысл неравенство (указывается направлением знака).

    Говорят, что два неравенства дают

    (а) тот же смысл , если знаки неравенства указывают в том же направлении; и

    (б) противоположный смысл , если признаки неравенства указывают в противоположном направлении.2-7 < 1`.

    Решение неравенства

    Решение неравенства состоит из всех значения переменной, которые делают неравенство истинным утверждение.

    Условные неравенства — это те, которые справедливы для некоторые, но не все значения переменной.

    Абсолютное неравенство — это неравенство, справедливое для всех значения переменной.

    Решение неравенства состоит только из действительных чисел в виде условия « меньше или больше » не определены для сложные числа.2+ 1> 0` верно для всех значений x и, следовательно, составляет абсолютное неравенство .

    Графическое представление неравенств

    Пример 5

    (a) Чтобы отобразить `x> 2` графически, мы используем открытый обведите кружком в точке `2` на числовой прямой и черту справа от нее точка со стрелкой вправо:

    Открытый кружок показывает, что точка не является частью указанное решение.

    (b) Для графического отображения `x ≤1` мы используем сплошную обведите 1 в числовой строке и линию слева от этого точка, со стрелкой, указывающей влево:

    Сплошной кружок показывает, что точка является частью указанного решение.

    (c) Чтобы обозначить «−2

    Теперь мы исследуем некоторые ключевые свойства неравенств.

    Свойство 1 — Сложение или вычитание числа

    смысл неравенства не меняется при добавлении или вычитании того же числа обе стороны неравенства.

    Пример 6

    Используя неравенство:

    `9> 6`

    прибавив `4` к обеим сторонам, получим

    `9 + 4> 6 ​​+ 4`

    то есть `13> 10`, что все еще верно

    вычитание 12 с каждой стороны оригинала дает

    `9 — 12> 6 — 12`

    то есть `-3> -6`, что все еще верно

    Свойство 2 — Умножение на положительное число

    смысл неравенства не изменяется, если обе стороны умножаются или делятся на такое же положительное число.

    Пример 7

    Используя неравенство:

    `8 <15`

    Умножение обеих сторон на `2` дает

    `8 × 2 <15 × 2`

    то есть `16 <30`, что все еще верно

    Если разделить обе стороны оригинала на `2`, получится

    `8/2 <15/2`

    то есть `4 <7,5`, что все еще верно

    Свойство 3 — Умножение на отрицательное число

    Смысл неравенства — в обратном порядке , если оба стороны умножаются или делятся на один и тот же минус количество.

    Пример 8

    Начнем с неравенства `4> −2`.

    Умножение обеих сторон на `-3` дает

    `4 × −3> -2 × −3`

    `-12> 6`, что не соответствует

    Следовательно, правильное решение должно быть

    `4> −2`

    `4 × −3 <−2 × −3`

    `−12 <6` (обратите внимание на изменение используемого знака)

    Аналогичным образом разделив обе части исходного неравенства на `−2`, получим

    `4> −2`

    `4 ÷ −2 <−2 ÷ −2`

    `-2 <1` (Обратите внимание на изменение используемого знака)

    Свойство 4 —

    n -я ​​мощность

    Если обе части неравенства положительны и n является положительным целым числом, то неравенство образованные n -й степенью или n -й корнем обеих сторон имеют тот же смысл , что и данное неравенство.2`

    то есть `81> 36`, что все еще верно

    Извлечение квадратного корня из каждой стороны дает

    `sqrt (9)> sqrt (6)`

    то есть `3> 2,45`, что все еще верно

    [ Примечание: `sqrt (9)` не равно `± 3`. По соглашению мы берем только положительный квадратный корень. См. Обсуждение на √16 — сколько ответов?]

    Упражнение

    Изобразите данное неравенство на номерная строка:

    `1

    Ответ

    Нам нужно иметь открытых круга для «1», поскольку он не включен, но закрытых кругов для «4», поскольку он включен.

    Неравенство

    Неравенство

    Содержание : Эта страница соответствует § 2.5 (стр. 216) текст.

    Предлагаемые задачи из текста:

    с. 225 # 11, 12, 13, 14, 16, 28, 33, 35, 38, 41, 53, 56, 62, 63, 68, 69

    Линейные неравенства

    Сочетания неравенств

    Неравенства, связанные с абсолютными значениями

    Полиномиальные неравенства

    Рациональные неравенства


    Линейные неравенства

    Неравенство — это сравнение выражений на «меньше» (<), «меньше или равно» на «(<=)», «больше» (>) или «больше или равно» (> =).Обратите внимание, что Html не поддерживает стандартные символы «меньше или равно» и «больше или равно», поэтому мы используем <= и> = для этих отношений.

    Пример 1 . х + 3 <= 10

    Решение для неравенства в x — это число, такое, что когда мы подставляем это число вместо x, мы имеем верное заявление. Итак, 4 — это решение, например 1, а 8 — нет. Набор решений неравенства набор всех решений.Обычно неравенство имеет бесконечно много решений, и множество решений легко описывается с использованием обозначения интервалов.

    Набор решений в примере 1 — это набор всех x <= 7. В интервальной записи это набор (-inf, 7], где мы используем inf для обозначения бесконечности.

    Линейное неравенство — это такое неравенство, что если бы мы заменили неравенство соотношением равенства, то мы имели бы линейное уравнение. Решение линейных неравенств очень похоже на решение линейных уравнений с одним важным отличием.

    Когда вы умножаете или делите обе стороны неравенства на отрицательное число, направление неравенство обращено вспять.

    Вы можете увидеть это, используя неравенство без переменных.

    Пример 2 .

    3 <7. Это ИСТИНА.

    (3) (- 2) <(7) (- 2). Это ЛОЖЬ, потому что -6 находится справа от -14 в числовой строке.Следовательно, -6> -14.

    (3) (- 2)> (7) (- 2). Это правда. Итак, когда мы умножаем исходное неравенство на -2, мы должны перевернуть направление, чтобы получить еще одно верное утверждение.

    Примечание : Как правило, мы не можем умножать или делить обе стороны неравенства на выражение с переменной, потому что некоторые значения переменной могут сделать выражение положительным, а некоторые — отрицательным.

    Пример 3 .

    7 — 2x <3.

    -2x <-4.

    х> 2.

    Примечание : Когда мы разделили обе стороны неравенства на -2, мы изменили направление неравенства.

    Посмотрите на графики функций по обе стороны от неравенства.

    Для выполнения неравенства 7 — 2x должно быть меньше 3. Итак, мы ищем такие числа x, что точка на графике y = 7 — 2x находится на ниже точки на графике y = 3.Это верно для x> 2. В интервале обозначение множество решений — (2, inf).

    Есть еще один способ использовать графическую утилиту для решения этого неравенства. В Java Grapher выражение (7-2 * x) L3 имеет значение 1 для чисел x, удовлетворяющих неравенству, и значение 0 для других чисел Икс. На рисунке ниже показан график (7-2 * x) L3, нарисованный Grapher.

    Упражнение 1:

    Решите неравенство 4 — x> 1 + 3x.Ответ

    Вернуться к содержанию

    Сочетания неравенств

    Пример 4 .

    Найдите все числа x такие, что -3 <5 - 2x и 5 — 2x <9.

    -3 <5 - 2x

    -8 <-2x

    4> x

    (-inf, 4)

    И

    5 — 2x <9

    -2x <4

    x> -2

    (-2, инф)

    Чтобы удовлетворить обоим неравенствам, число должно быть в обоих наборах решений.Итак, числа, удовлетворяющие обоим неравенства — это значения на пересечении двух наборов решений, которые представляют собой набор (-2, 4) в интервале обозначение.

    Задача выше обычно записывается как двойное неравенство .

    -3 <5 - 2x <9 означает -3 <5 - 2x и 5 — 2x <9.

    Примечание: Когда мы решили два неравенства по отдельности, шаги в двух задачах были тем же.Следовательно, для одновременного решения неравенств можно использовать обозначение двойного неравенства.

    -3 <5 - 2x <9.

    -8 <-2x <4.

    4> х> -2.

    В терминах графиков эта задача соответствует нахождению таких значений x, что соответствующая точка на график y = 5 — 2x находится между графиками y = -3 и y = 9.

    Пример 5 .

    Найдите все числа x такие, что x + 1 <0 или x + 1> 3.

    В примере 4 выше мы искали числа, удовлетворяющие обоим неравенствам. Здесь мы хотим найти числа удовлетворяющие любому из неравенств. Это соответствует объединению наборов решений вместо пересечения.

    Не используйте в этой ситуации обозначение двойного неравенства.

    x + 1 <0

    x <-1

    (-inf, -1)

    ИЛИ x + 1> 3

    x> 2

    (2, инф)

    Набор решений — это объединение двух интервалов (-inf, -1) и (2, inf).

    Упражнение 2:

    (а) 1 <3 + 5x <7 Ответ

    (b) 2 — x <1 или 2 - x> 5 Ответ

    Вернуться к содержанию

    Неравенства, связанные с абсолютными значениями

    Неравенства, включающие абсолютные значения, можно переписать как комбинации неравенств.

    Пусть a будет положительным числом.

    | x |

    | x | > a тогда и только тогда, когда x <-a или x> a.

    Чтобы понять эти утверждения, подумайте о числовой прямой. Абсолютное значение числа — это расстояние номер начинается с 0 в числовой строке. Итак, неравенство | x |

    Неравенство | x | > a удовлетворяют числа, расстояние от которых до 0 больше, чем a.Это означает числа которые либо больше, либо меньше -a.

    Пример 6 .

    | 3 + 2x | <= 7.

    -7 <= 3 + 2x <= 7.

    -10 <= 2x <= 4.

    -5 <= x <= 2.

    x находится в [-5, 2].

    С точки зрения графиков, мы ищем такие значения x, что соответствующая точка на графике y = | 3+ 2x | либо ниже, либо равна точке на графике y = 7.

    Пример 7 .

    | 5 — 2x | > 3.

    5 — 2x <-3 или 5 - 2x> 3.

    -2x <-8 или -2x> -2.

    x> 4 или x <1.

    x находится в (4, inf) union (-inf, 1).

    Этот набор решений соответствует области, где график y = | 5 — 2x | находится над графиком y = 3.

    Упражнение 3 :

    Решите следующие неравенства.Используйте графическую утилиту, чтобы проверить свои ответы.

    (а) | 3 + х | <4.

    (б) | 2 — х | > 3.

    Вернуться к содержанию

    Полиномиальные неравенства

    Пример 8 .

    x 2 — x — 6 <0.

    Первый шаг — найти нули многочлена x 2 — x — 6.

    х 2 — х — 6 = 0.

    (x + 2) (x — 3) = 0.

    x = -2 или x = 3.

    -2 и 3 называются критическими числами неравенства.

    Примечание: -2 и 3 не входят в набор решений неравенства. Мы ищем ценности x, где многочлен отрицательный. Множество решений неравенства соответствует области, в которой граф полинома ниже оси абсцисс. Критические числа -2 и 3 — это места пересечения графика ось абсцисс.

    Критические числа делят ось абсцисс на три интервала, называемых контрольными интервалами для неравенства.

    Тестовые интервалы: (-inf, -2), (-2, 3), (3, inf).

    Мы собираемся использовать тот факт, что полиномиальные функции непрерывны . Это означает, что их графики не делать перерывов и прыжков.

    Так как мы нашли все пересечения по оси x графика x 2 — x — 6 на протяжении каждого интервала тестирования график должен быть либо выше оси x, либо ниже нее.Здесь нам нужно знать, что на графике нет никаких перерывов. Это означает, что мы можем выбрать любое число, которое нам нравится в тестовом интервале, и оценить полином на это число, чтобы увидеть, находится ли график выше или ниже оси абсцисс на протяжении всего интервала тестирования.

    (-inf, -2): -5 находится в интервале. (-5) 2 — (-5) — 6 = 24> 0, поэтому график y = x 2 — x — 6 находится над осью x на всем интервале (-inf, -2).

    (-2; 3): 0 находится в интервале.0 2 -0-6 = -6 <0, поэтому график y = x 2 — x — 6 находится ниже оси абсцисс на всем интервале.

    (3, inf): 4 находится в интервале. 4 2 — 4-6 = 6> 0, поэтому график y = x 2 — x — 6 находится над осью абсцисс на всем интервале.

    Поскольку мы ищем области, где график находится ниже оси, набор решений равен -2

    Распространенная ошибка

    Мы будем использовать проблему из примера 8, чтобы проиллюстрировать распространенную ошибку.

    x 2 — x — 6 <0.

    (x + 2) (x — 3) <0 Хорошо до этого момента.

    x + 2 <0 или x - 3 <0 НЕПРАВИЛЬНО!

    Если произведение двух чисел равно , равному : 0, то хотя бы одно из чисел должно быть 0. Однако произведение двух отрицательных чисел не является отрицательным, поэтому этот подход бесполезен для решения неравенств.

    Пример 9 .

    1.2 x 3 + 3,07 x 2 — x — 3,71> 0.

    Эта задача намного сложнее неравенства в предыдущем примере! Фактор непросто, поэтому мы не сможем найти точные значения критических чисел. Мы будем использовать графическую утилиту, чтобы приблизить критические числа. График полинома показан ниже.

    y = 1,2 x 3 + 3,07 x 2 — x — 3,71

    Критические числа примерно -2.35, -1,25 и 1,05. В этой задаче мы ищем регионы, где график находится над осью.

    Набор решений : (-2.35, -1.25) union (1.05, inf).

    Упражнение 4 :

    Решите неравенство x 2 + 3x — 4> 0. Используйте графическую утилиту, чтобы проверить свое решение.

    Вернуться к содержанию

    Рациональные неравенства

    Рациональное выражение — это одна из форм полинома, деленная на полином.В общем, графики рациональных функций есть перерывы. Они не определены в нулях знаменателя. Это единственные места, где есть ломается, поэтому мы можем использовать ту же технику для решения рациональных неравенств, которую мы используем для полиномиальных неравенств.

    Пример 10 .

    Критические числа для рационального неравенства — это все нули числителя и знаменателя. С числитель и знаменатель уже учтены в этом примере, мы видим, что критических числа являются -3, 5 и 1.

    Три критических числа делят числовую строку на четыре тестовых интервала.

    (-inf, -3): -4 находится в интервале, а рациональная функция, оцененная как -4, равна -9/15. Поскольку значение отрицательное, график рациональной функции находится на ниже оси абсцисс на всем интервале.

    (-3, 1): 0 находится в интервале. Значение функции в 0 равно 5, что положительно. График функции на выше по оси абсцисс во всем интервале.

    (1, 5): 2 находится в интервале. Значение 2 равно -5. График функции расположен на ниже по оси абсцисс.

    (5, inf): 6 находится в интервале. Значение 6 — 9/15. График функции расположен на выше по оси абсцисс.

    Мы ищем области, в которых график находится выше оси x, поэтому набор решений равен (-3, 1) union (5, инф.).

    Примечание: Можно использовать графическую утилиту, чтобы увидеть, с какой стороны оси x находится график над различные интервалы тестирования.В некоторых случаях вам необходимо решить алгебраически, чтобы найти точные значения критических чисел, но как только это будет сделано, график предоставит быстрый способ решить проблему.

    График y = (x + 3) (x — 5) / 3 (x — 1)

    При работе с неравенством следует помнить о двух важных моментах:

    1. Нам нужно сравнить выражение с 0. Итак, если мы начнем с задачи

    x 2 — 3x — 11

    x 2 — 4x — 21 <0.

    2. Не умножайте обе части неравенства на выражение с переменной.

    Например, учитывая проблему, сделать not умножьте обе части на x. Правильный способ решения этой проблемы следующий:

    Теперь мы видим, что критические числа равны 0 (из знаменателя), 1 и -1.

    Упражнение 5 :

    (a) Завершите решение x 2 — 3x — 11

    (b) Завершите решение и проверьте свое решение. с помощью графической утилиты.

    Вернуться к содержанию


    Алгебра — линейные неравенства

    Показать уведомление для мобильных устройств Показать все заметки Скрыть все заметки

    Похоже, вы используете устройство с «узкой» шириной экрана ( i.е. вы, вероятно, пользуетесь мобильным телефоном). Из-за особенностей математики на этом сайте лучше всего просматривать в ландшафтном режиме. Если ваше устройство не находится в альбомном режиме, многие уравнения будут отображаться сбоку от вашего устройства (вы сможете прокручивать их, чтобы увидеть их), а некоторые пункты меню будут обрезаны из-за узкой ширины экрана.

    Раздел 2-11: Линейные неравенства

    До этого момента в этой главе мы сосредоточились на решении уравнений.Пришло время немного переключиться и начать думать о решении проблемы неравенства. Прежде чем мы перейдем к решению неравенств, мы должны сначала рассмотреть пару основных моментов.

    На данном этапе вашей математической карьеры предполагается, что вы знаете, что

    \ [a

    означает, что \ (a \) — некоторое число, строго меньшее, чем \ (b \). Также предполагается, что вы знаете, что

    \ [а \ ге б \]

    означает, что \ (a \) — некоторое число, которое либо строго больше, чем \ (b \), либо точно равно \ (b \).Точно так же предполагается, что вы знаете, что делать с двумя оставшимися неравенствами. > (больше) и \ (\ le \) (меньше или равно).

    Мы хотим обсудить некоторые проблемы с обозначениями и некоторые тонкости, которые иногда возникают у студентов, когда они действительно начинают работать с неравенством.

    Во-первых, помните, что когда мы говорим, что \ (a \) меньше \ (b \), мы имеем в виду, что \ (a \) находится слева от \ (b \) на числовой строке. Итак,

    \ [- 1000

    — истинное неравенство.

    Затем не забывайте, как правильно интерпретировать \ (\ le \) и \ (\ ge \). Оба следующих утверждения являются истинными неравенствами.

    \ [4 \ le 4 \ hspace {0,25 дюйма} \ hspace {0,25 дюйма} \ hspace {0,25 дюйма} — 6 \ le 4 \]

    В первом случае 4 равно 4 и, следовательно, «меньше или равно» 4. Во втором случае -6 строго меньше 4 и, таким образом, «меньше или равно» 4. Наиболее часто встречается ошибка состоит в том, чтобы решить, что первое неравенство не является истинным неравенством. Также будьте осторожны, чтобы не принять эту интерпретацию и не перевести ее на <и / или>.Например,

    \ [4

    не является истинным неравенством, поскольку 4 равно 4, а не строго меньше 4.

    Наконец, в этом и последующих разделах мы увидим множество двойных неравенств , так что мы не можем забыть о них. Следующее — двойное неравенство.

    \ [- 9

    В двойном неравенстве мы говорим, что оба неравенства должны выполняться одновременно. В этом случае 5 определенно больше -9 и в то же время меньше или равно 6.Следовательно, это двойное неравенство является истинным неравенством.

    С другой стороны,

    \ [10 \ le 5

    не является истинным неравенством. Хотя верно, что 5 меньше 20 (так что второе неравенство верно), неверно, что 5 больше или равно 10 (поэтому первое неравенство неверно). Если хотя бы одно из неравенств в двойном неравенстве неверно, то все неравенство неверно. Этот момент более важен, чем вы можете себе представить.В следующем разделе мы столкнемся с ситуациями, когда многие студенты пытаются объединить два неравенства в двойное неравенство, которое просто невозможно объединить, так что будьте осторожны.

    Следующая тема, которую нам нужно обсудить, — это идея обозначения интервала . Обозначение интервалов — это очень хорошее обозначение неравенств, которое будет широко использоваться в следующих нескольких разделах этой главы.

    Наилучший способ определения обозначения интервалов — это следующая таблица.В таблице три столбца. Каждая строка содержит неравенство, график, представляющий неравенство, и, наконец, обозначение интервала для данного неравенства.

    Помните, что квадратные скобки «[» или «]» означают, что мы включаем конечную точку, а круглые скобки «(» или «)» означают, что мы не включаем конечную точку.

    Итак, с первыми четырьмя неравенствами в таблице обозначение интервалов на самом деле представляет собой не что иное, как график без числовой прямой.С последними четырьмя неравенствами обозначение интервала — это почти график, за исключением того, что нам нужно добавить соответствующую бесконечность, чтобы убедиться, что мы получаем правильную часть числовой прямой. Также обратите внимание, что бесконечности НИКОГДА не получают скобки. У них есть только круглые скобки.

    Прежде чем перейти к решению неравенств, необходимо сделать последнее замечание об обозначении интервалов. Всегда помните, что когда мы записываем интервальное обозначение для неравенства, число слева должно быть меньшим из двух.

    Пришло время подумать о решении линейных неравенств. При решении неравенств мы будем использовать следующий набор фактов. Обратите внимание, что факты приведены для <. Однако мы можем записать эквивалентный набор фактов для оставшихся трех неравенств.

    1. Если \ (a
    2. Если \ (a 0 \), то \ (ac
    3. Если \ (a bc \) и \ (\ frac {a} {c}> \ frac {b} {c} \). В этом случае, в отличие от предыдущего факта, если \ (c \) отрицательно, нам нужно изменить направление неравенства, когда мы умножаем или делим обе части неравенства на \ (c \).

    Это почти те же факты, которые мы использовали для решения линейных уравнений. Единственное реальное исключение — третий факт. Это важный факт, поскольку он часто используется неправильно и / или часто забывается при решении проблемы неравенства.

    Если вы не уверены, что полагаете, что знак \ (c \) имеет значение для второго и третьего фактов, рассмотрите следующий пример числа.

    \ [- 3

    Я надеюсь, что мы все согласимся, что это истинное неравенство.Теперь умножьте обе стороны на 2 и -2.

    \ [\ begin {align *} — 3 и 5 \ left ({- 2} \ right) \\ — 6 & — 10 \ end {align *} \]

    Конечно, при умножении на положительное число направление неравенства остается неизменным, однако при умножении на отрицательное число направление неравенства меняется.

    Хорошо, давайте устраним некоторые неравенства. Мы начнем с неравенств, в которых есть только одно неравенство.Другими словами, мы отложим решение двойных неравенств для следующего набора примеров.

    Здесь мы должны помнить, что мы просим определить все значения переменной, которые мы можем подставить в неравенство и получить истинное неравенство. Это означает, что наши решения в большинстве случаев сами по себе будут неравенствами.

    Пример 1 Решение следующих неравенств. Приведите как неравенство, так и интервальную форму записи решения.
    1. \ (- 2 \ влево ({m — 3} \ right) <5 \ left ({m + 1} \ right) - 12 \)
    2. \ (2 \ left ({1 — x} \ right) + 5 \ le 3 \ left ({2x — 1} \ right) \)
    Показать все решения Скрыть все решения Показать обсуждение

    Решение отдельных линейных неравенств во многом повторяет процесс решения линейных уравнений. Мы упростим обе стороны, получим все члены с переменной с одной стороны и числа с другой стороны, а затем умножим / разделим обе стороны на коэффициент переменной, чтобы получить решение.Вы должны помнить одну вещь: если вы умножаете / делите на отрицательное число, то меняете направление неравенства.


    a \ (- 2 \ left ({m — 3} \ right) Показать решение

    Здесь действительно особо нечего делать, кроме как следовать описанному выше процессу.

    \ [\ begin {align *} — 2 \ left ({m — 3} \ right) & \ frac {{13}} {7} \ end {align *} \]

    Вы уловили тот факт, что направление неравенства здесь изменилось, не так ли? Мы разделились на «-7» и нам пришлось менять направление.Неравенство решения имеет вид \ (m> \ frac {{13}} {7} \). Обозначение интервала для этого решения: \ (\ left ({\ frac {{13}} {7}, \ infty} \ right) \).


    b \ (2 \ left ({1 — x} \ right) + 5 \ le 3 \ left ({2x — 1} \ right) \) Показать решение

    Опять же, здесь особо нечего делать.

    \ [\ begin {align *} 2 \ left ({1 — x} \ right) + 5 & \ le 3 \ left ({2x — 1} \ right) \\ 2 — 2x + 5 & \ le 6x — 3 \\ 10 & \ le 8x \\ \ frac {{10}} {8} & \ le x \\ \ frac {5} {4} & \ le x \ end {align *} \]

    Теперь, с этим неравенством, мы закончили с переменной с правой стороны, когда это более традиционно с левой стороны.Итак, давайте поменяем местами, чтобы переменная оказалась слева. Обратите внимание, однако, что нам нужно будет также изменить направление неравенства, чтобы убедиться, что мы не изменим ответ. Итак, вот обозначение неравенства для неравенства.

    \ [x \ ge \ frac {5} {4} \]

    Обозначение интервала для решения: \ (\ left [{\ frac {5} {4}, \ infty} \ right) \).

    Теперь давайте решим несколько двойных неравенств.Этот процесс в некотором смысле похож на решение отдельных неравенств, но в остальном сильно отличается. Поскольку существует два неравенства, невозможно получить переменные с «одной стороны» неравенства и числа с другой. Легче увидеть, как это работает, если мы рассмотрим пару примеров, так что давайте сделаем это.

    Пример 2 Решите каждое из следующих неравенств. Приведите для решения формы обозначений неравенства и интервалов.
    1. \ (- 6 \ le 2 \ left ({x — 5} \ right) <7 \)
    2. \ (- 3 <\ frac {3} {2} \ left ({2 - x} \ right) \ le 5 \)
    3. \ (- 14 <- 7 \ влево ({3x + 2} \ вправо) <1 \)
    Показать все решения Скрыть все решения a \ (- 6 \ le 2 \ left ({x — 5} \ right) Показать решение

    Процесс здесь довольно похож на процесс для одиночных неравенств, но сначала нам нужно быть осторожными в нескольких местах.Нашим первым шагом в этом случае будет удаление скобок в среднем члене.

    \ [- 6 \ le 2x — 10

    Теперь мы хотим, чтобы \ (x \) был сам по себе в среднем члене и только числа в двух внешних членах. Для этого мы будем добавлять / вычитать / умножать / делить по мере необходимости. Единственное, что нам нужно здесь помнить, это то, что если мы делаем что-то для среднего срока, нам нужно сделать то же самое с ОБЕИМ из исходящих терминов. Одна из наиболее частых ошибок на этом этапе — добавить что-то, например, в середину, и добавить это только к одной из двух сторон.

    Хорошо, мы прибавим 10 ко всем трем частям, а затем разделим все три части на две.

    \ [\ begin {array} {c} 4 \ le 2x

    Это форма ответа неравенства. Ответ в виде интервальной записи \ (\ left [{2, \ frac {{17}} {2}} \ right) \).


    b \ (- 3 Показать решение

    В этом случае первое, что нам нужно сделать, это очистить дроби, умножив все три части на 2. Затем мы продолжим, как и в первой части.

    \ [\ begin {array} {c} — 6

    На этом мы еще не закончили, но нам нужно быть очень осторожными на следующем шаге. На этом этапе нам нужно разделить все три части на -3. Однако напомним, что всякий раз, когда мы делим обе стороны неравенства на отрицательное число, нам нужно изменить направление неравенства. Для нас это означает, что оба неравенства должны изменить направление здесь.

    \ [4> х \ ge — \ frac {4} {3} \]

    Итак, существует неравномерная форма решения.Нам нужно будет быть осторожным с обозначением интервалов для решения. Во-первых, обозначение интервала НЕ \ (\ left ({4, — \ frac {4} {3}} \ right] \). Помните, что в обозначении интервала меньшее число всегда должно располагаться слева! правильное обозначение интервала для решения: \ (\ left [{- \ frac {4} {3}, 4} \ right) \).

    Также обратите внимание, что это также соответствует неравенству формы решения. Неравенство говорит нам, что \ (x \) — это любое число от 4 до \ (- \ frac {4} {3} \) или, возможно, само \ (- \ frac {4} {3} \), и это в точности что нам говорят обозначения интервалов.

    Кроме того, неравенство можно перевернуть, чтобы получить меньшее число слева, если мы захотим. Вот та форма,

    \ [- \ frac {4} {3} \ le x

    При этом не забудьте также правильно обработать неравенства.


    c \ (- 14 Показать решение

    Не особо для этого. Мы продолжим так же, как и в предыдущих двух.

    \ [\ begin {array} {c} — 14

    Не волнуйтесь, что одна из сторон теперь равна нулю.Это не проблема. Опять же, как и в предыдущей части, мы будем делить на отрицательное число, поэтому не забудьте изменить направление неравенств.

    \ [\ begin {array} {c} \ displaystyle 0> x> — \ frac {{15}} {{21}} \\ \ displaystyle 0> x> — \ frac {5} {7} \ hspace {0,25 in} {\ mbox {OR}} \ hspace {0,25in} — \ frac {5} {7}

    Для решения подойдет любое из неравенств во второй строке. Интервальное обозначение решения — \ (\ left ({- \ frac {5} {7}, 0} \ right) \).

    При решении двойных неравенств обязательно обратите внимание на неравенства, которые есть в исходной задаче. Одна из наиболее распространенных ошибок здесь — начать с задачи, в которой одно из неравенств имеет вид <или>, а другое — \ (\ le \) или \ (\ ge \), как мы делали в первых двух частях в предыдущем примере, а затем в окончательном ответе они оба являются <или>, либо они оба являются \ (\ le \) или \ (\ ge \). Другими словами, легко сделать оба неравенства одинаковыми.Будьте осторожны с этим.

    Есть еще один последний пример, над которым мы хотим работать.

    Пример 3 Если \ (- 1 Показать решение

    Это проще, чем может показаться на первый взгляд. Все, что мы действительно собираемся сделать, это начать с данного неравенства, а затем манипулировать средним членом, чтобы он выглядел как второе неравенство. Опять же, нам нужно помнить, что все, что мы делаем со средним термином, нам также необходимо сделать с двумя внешними членами.

    Итак, сначала умножим все на 2.

    \ [- 2

    Теперь прибавьте 3 ко всему.

    \ [1

    Теперь у нас есть средний член, идентичный второму неравенству в постановке задачи, и поэтому все, что нам нужно сделать, это выбрать \ (a \) и \ (b \). Из этого неравенства видно, что \ (a = 1 \) и \ (b = 11 \).

    .

    Добавить комментарий

    Ваш адрес email не будет опубликован. Обязательные поля помечены *